Cuestiones
ayuda
option
Mi Daypo

TEST BORRADO, QUIZÁS LE INTERESEGeneral

COMENTARIOS ESTADÍSTICAS RÉCORDS
REALIZAR TEST
Título del test:
General

Descripción:
Generalidades

Autor:
AV Test
(Otros tests del mismo autor)

Fecha de Creación:
18/12/2014

Categoría:
Otros

Número preguntas: 817
Comparte el test:
Facebook
Twitter
Whatsapp
Comparte el test:
Facebook
Twitter
Whatsapp
Últimos Comentarios
No hay ningún comentario sobre este test.
Temario:
The primary reason the angle of attack must be increased, to maintain a constant altitude during a coordinated turn, is because the * Thrust is acting in a different direction, causing a reduction in airspeed and loss of lift. Vertical component of lift has decreased as the result of the bank. Use of ailerons has increased the drag.
Which is true regarding the use of flaps during level turns? * The lowering of flaps increases the stall speed. The raising of flaps increases the stall speed. Raising flaps will require added forward pressure on the yoke or stick.
Defines VNO as * Maximum operating limit speed. Maximum structural cruising speed. Never-exceed speed.
Airplane wing loading during a level coordinated turn in smooth air depends upon the ** Rate of turn. Angle of bank. True airspeed.
The stalling speed of an airplane is most affected by * Changes in air density Variations in flight altitude. Variations in airplane loading.
During the transition from straight-and-level flight to a climb, the angle of attack is increased and lift * Is momentarily decreased. Remains the same. Is momentarily increased.
Load factor is the lift generated by the wings of an aircraft at any given time * Divided by the total weight of the aircraft. Multiplied by the total weight of the aircraft. Divided by the basic empty weight of the aircraft.
Lift on a wing is most properly defined as the * Force acting perpendicular to the relative wind. Differential pressure acting perpendicular to the chord of the wing. Reduced pressure resulting from a laminar flow over the upper camber of an airfoil, which acts perpendicular to the mean camber.
While holding the angle of bank constant in a level turn, if the rate of turn is varied the load factor would * Remain constant regardless of air density and the resultant lift vector. Vary depending upon speed and air density provided the resultant lift vector varies proportionately. Vary depending upon the resultant lift vector.
Which is true regarding the forces acting on an aircraft in a steady-state descent? The sum of all * Upward forces is less than the sum of all downward forces. Forces is greater than the sum of all forward forces. Forward forces is equal to the sum of all rearward forces.
What performance is characteristic of flight at maximum lift/drag ratio in a propeller-driven airplane? Maximum * Gain in altitude over a given distance. Range and maximum distance glide. Coefficient of lift and minimum coefficient of drag.
The need to slow an aircraft below VA is brought about by the following weather phenomenon: * High density altitude which increases the indicated stall speed. Turbulence which causes an increase in stall speed. Turbulence which causes a decrease in stall speed.
In theory, if the airspeed of an airplane is doubled while in level flight, parasite drag will become * Twice as great. Half as great. Four times greater.
As airspeed decreases in level flight below that speed for maximum lift/drag ratio, total drag of an airplane * Decreases because of lower parasite drag. Increases because of increased induced drag. Increases because of increased parasite drag.
If the airspeed is increased from 90 knots to 135 knots during a level 60° banked turn, the load factor will * Increase as well as the stall speed. Decrease and the stall speed will increase. Remain the same but the radius of turn will increase.
The angle of attack at which a wing stalls remains constant regardless of * Weight, dynamic pressure, bank angle, or pitch attitude. Dynamic pressure, but varies with weight, bank angle, and pitch attitude. Weight and pitch attitude, but varies with dynamic pressure and bank angle.
Which statement is true, regarding the opposing forces acting on an airplane in steady-state level flight? * These forces are equal. Thrust is greater than drag and weight and lift are equal. Thrust is greater than drag and lift is greater than weight.
On a wing, the force of lift acts perpendicular to, and the force of drag acts parallel to the * Chord line. Flightpath. Longitudinal axis.
An aircraft wing is designed to produce lift resulting from a difference in the * Negative air pressure below and a vacuum above the wing's surface. Vacuum below the wing's surface and greater air pressure above the wing's surface. Higher air pressure below the wing's surface and lower air pressure above the wing's surface.
In theory, if the angle of attack and other factors remain constant and the airspeed is doubled, the lift produced at the higher speed will be * The same as at the lower speed. Two times greater than at the lower speed. Four times greater than at the lower speed.
By changing the angle of attack of a wing, the pilot can control the airplane's * Lift, airspeed, and drag. Lift, airspeed, and CG. Lift and airspeed, but not drag.
A rectangular wing, as compared to other wing planforms, has a tendency to stall first at the * Wingtip, with the stall progression toward the wing root. Wing root, with the stall progression toward the wing tip. Center trailing edge, with the stall progression outward toward the wing root and tip.
Stall speed is affected by * Weight, load factor, and power. Load factor, angle of attack, and power. Angle of attack, weight, and air density.
Which statement is true relative to changing angle of attack? * A decrease in angle of attack will increase pressure below the wing, and decrease drag. An increase in angle of attack will decrease pressure below the wing, and increase drag. An increase in angle of attack will increase drag.
For a given angle of bank, in any airplane, the load factor imposed in a coordinated constant-altitude turn * Is constant and the stall speed increases. Varies with the rate of turn. Is constant and the stall speed decreases.
If severe turbulence is encountered during flight, the pilot should reduce the airspeed to * Minimum control speed. Design-maneuvering speed. Maximum structural cruising speed.
A propeller rotating clockwise as seen from the rear, creates a spiraling slipstream. The spiraling slipstream, along with torque effect, tends to rotate the airplane to the * Right around the vertical axis, and to the left around the longitudinal axis. Left around the vertical axis, and to the right around the longitudinal axis. Left around the vertical axis, and to the left around the longitudinal axis.
Which is true regarding the force of lift in steady, unaccelerated flight? * At lower airspeeds the angle of attack must be less to generate sufficient lift to maintain altitude. There is a corresponding indicated airspeed required for every angle of attack to generate sufficient lift to maintain altitude. An airfoil will always stall at the same indicated airspeed, therefore, an increase in weight will require an increase in speed to generate sufficient lift to maintain altitude.
The ratio between the total airload imposed on the wing and the gross weight of an aircraft in flight is known as * Load factor and directly affects stall speed. Aspect load and directly affects stall speed. Load factor and has no relation with stall speed.
The vertical line from point D to point G is represented on the airspeed indicator by the maximum speed limit of the * Green arc. Yellow arc. White arc.
If an aircraft with a gross weight of 2,000 pounds was subjected to a 60° constant-altitude bank, the total load would be * 3,000 pounds. 4,000 pounds. 12,000 pounds.
If the airplane attitude initially tends to return to its original position after the elevator control is pressed forward and released, the airplane displays * Positive dynamic stability. Positive static stability. Neutral dynamic stability.
Longitudinal stability involves the motion of the airplane controlled by its * Rudder. Elevator. Ailerons.
Longitudinal dynamic instability in an airplane can be identified by * Bank oscillations becoming progressively steeper. Pitch oscillations becoming progressively steeper. Trilatitudinal roll oscillations becoming progressively steeper.
If the airplane attitude remains in a new position after the elevator control is pressed forward and released, the airplane displays * Neutral longitudinal static stability. Positive longitudinal static stability. Neutral longitudinal dynamic stability.
In a rapid recovery from a dive, the effects of load factor would cause the stall speed to * Increase. Decrease. Not vary.
When turbulence is encountered during the approach to a landing, what action is recommended and for what primary reason? * Increase the airspeed slightly above normal approach speed to attain more positive control. Decrease the airspeed slightly below normal approach speed to avoid overstressing the airplane. Increase the airspeed slightly above normal approach speed to penetrate the turbulence as quickly as possible.
Recovery from a stall in any airplane becomes more difficult when its * Center of gravity moves forward. Elevator trim is adjusted nosedown. Center of gravity moves aft.
Why is it necessary to increase back elevator pressure to maintain altitude during a turn? To compensate for the * Loss of the vertical component of lift. Loss of the horizontal component of lift and the increase in centrifugal force. Rudder deflection and slight opposite aileron throughout the turn.
If airspeed is increased during a level turn, what action would be necessary to maintain altitude? The angle of attack * And angle of bank must be decreased. Must be increased or angle of bank decreased. Must be decreased or angle of bank increased.
Name the four fundamentals involved in maneuvering an aircraft. * Power, pitch, bank, and trim. Thrust, lift, turns, and glides. Straight-and-level flight, turns, climbs, and descents.
A pilot's most immediate and vital concern in the event of complete engine failure after becoming airborne on takeoff is * Maintaining a safe airspeed. Landing directly into the wind. Turning back to the takeoff field.
To increase the rate of turn and at the same time decrease the radius, a pilot should * Maintain the bank and decrease airspeed. Increase the bank and increase airspeed. Increase the bank and decrease airspeed.
(Refer to figure 2.) Select the correct statement regarding stall speeds. The airplane will stall * 10 knots higher in a power-on, 60° bank, with gear and flaps up, than with gear and flaps down. 25 knots lower in a power-off, flaps-up, 60° bank, than in a power-off, flaps-down, wings-level configuration. 10 knots higher in a 45° bank, power-on stall, than in a wings-level stall with flaps up.
An airplane leaving ground effect will * Experience a reduction in ground friction and require a slight power reduction. Experience an increase in induced drag and require more thrust. Require a lower angle of attack to maintain the same lift coefficient.
One of the main functions of flaps during the approach and landing is to * Decrease the angle of descent without increasing the airspeed. Provide the same amount of lift at a slower airspeed. Decrease lift, thus enabling a steeper-than-normal approach to be made.
The spoilers should be in what position when operating in a strong wind? * Extended during both a landing roll or ground operation. Retracted during both a landing roll or ground operation. Extended during a landing roll, but retracted during a ground operation.
The purpose of wing spoilers is to decrease * The drag. Landing speed. The lift of the wing.
Propeller efficiency is the * Ratio of thrust horsepower to brake horsepower. Actual distance a propeller advances in one revolution. Ratio of geometric pitch to effective pitch.
In order to achieve single engine climb performance with an engine failed in most twin prop aircraft, it is essential to: * Apply full power to the live engine (firewall) Minimize drag (flaps, gear, feather prop) Secure the failed engine.
If the same angle of attack is maintained in ground effect as when out of ground effect, lift will * increase,and induced drag will decrease. decrease, and parasite drag will increase. increase, and induced drag will increase.
In order to achieve a zero sideslip condition with a failed engine on a propeller aircraft, the pilot must: * Bank away from the live engine up to 5º Center the ball on the turn and slip indicator Bank into the live engine slightly.
When dealing with aerodynamics, VMC refers to: * Visual Meteorological Conditions Minimum airspeed at which directional control can be maintained with the critical engine inoperative Maximum airspeed at which an engine can be operated with one engine.
On an aircraft without counter-rotating props, the left engine is considered to be the critical one to fail because of the following factors: * Zero sideslip condition, engine wind milling and aft legal C of G. P-factor, Accelerated slipstream, Spiraling slipstream and Torque No power available Vs power required, yaw towards the left engine and sideslip.
Every physical process of weather is accompanied by or is the result of * A heat exchange. The movement of air. A pressure differentia.
What causes wind? * The Earth’s rotation. Air mass modification. Pressure differences.
In the Northern Hemisphere, the wind is deflected to the * Right by Coriolis force. Right by surface friction. Left by Coriolis force.
Why does the wind have a tendency to flow parallel to the isobars above the friction level? * Coriolis force tends to counterbalance the horizontal pressure gradient. Coriolis force acts perpendicular to a light connecting the highs and lows. Friction of the air with the Earth deflects the air perpendicular to the pressure gradient.
What prevents air from flowing directly from high-pressure areas to low-pressure areas? * Coriolis force. Surface friction. Pressure gradient force.
Which would increase the stability of an air mass? * Warning from below. Cooling from below. Decrease in water vapor.
Which feature is associated with the tropopause? * Constant height above the Earth. Abrupt change in the temperature lapse rate. Absolute upper limit of cloud formation.
A common location of clear air turbulence is * In an upper trough on the polar side of a jet stream. Near a ridge aloft on the equatorial side of a high-pressure flow. South of an east/west oriented high-pressure ridge in its dissipating stage.
The jet stream and associated clear air turbulence can sometimes be visually identified in flight by * Dust or haze at flight level. Long streaks of cirrus clouds. A constant outside air temperature.
During the winter months in the middle latitudes, the jet stream shifts toward the * North and speed decreases. South and speed increases. North and speed increases.
Which type of jet stream can be expected to cause the greater turbulence? * A straight jet stream associated with a low-pressure trough. A curving jet stream associated with a deep low-pressure trough. A jet stream occurring during the summer at the lower latitudes.
Which conditions are favorable for the formation of a surface based temperature inversion? * Clear, cool nights with calm or light wind. Area of unstable air rapidly transferring heat from the surface. Broad areas of cumulus clouds with smooth, level bases at the same altitude.
The wind system associated with a low-pressure area in the Northern Hemisphere is * An anticyclone and is caused by descending cold air. A cyclone and is caused by Coriolis force. An anticyclone and is caused by Coriolis force.
Which is the true with a respect to a high- or low-pressure system? * A high-pressure area or ridge is an area of rising air. A low-pressure area or trough is an area of descending air. A high-pressure area or ridge is an area of descending air.
Which is the true regarding high- or low-pressure systems? * A high-pressure area or ridge is an area of rising air. A low-pressure area or trough is an area of rising air. Both high- and low-pressure areas are characterized by descending air.
Which is true regarding actual air temperature and dew point temperature spread? The temperature spread * Decreases as the relative humidity decreases. Decreases as the relative humidity increases. Increases as the relative humidity increases.
Moisture is added to a parcel of air by * Sublimation and condensation. Evaporation and condensation. Evaporation and sublimation.
Which would decrease the stability of an air mass? * Warming from below. Cooling from below. Decrease in water vapor.
What determines the structure or type of clouds which will form as a result of air being forced to ascend? * The method by which the air is lifted. The stability of the air before lifting occurs. The relative humidity of the air after lifting occurs.
When conditionally unstable air with high-moisture content and very warm surface temperature is forecast, one can expect what type of weather? * Strong updrafts and stratonimbus clouds. Restricted visibility near the surface over a large area. Strong updrafts and cumulonimbus clouds.
If clouds form as a result of very stable, moist air being forced to ascend a mountain slope, the clouds will be * Cirrus type with no vertical development or turbulence. Cumulus type with considerable vertical development and turbulence. Stratus type with little vertical development and little or no turbulence.
What type weather can one expect from moist, unstable air, and very warm surface temperatures? * Fog and low stratus clouds. Continuous heavy precipitation. Strong updrafts and cumulonimbus clouds.
The conditions necessary for the formation of stratiform clouds are a lifting action and * Unstable, dry air. Stable, moist air. Unstable, moist air.
Which cloud types would indicate convective turbulence? * Cirrus clouds. Nimbostratus clouds. Towering cumulus clouds.
Which combination of weather-producing variables would likely result in cumuliform-type clouds, good visibility, and showery rain? * Stable, moist air and orographic lifting. Unstable, moist air and orographic lifting. Unstable, moist air and no lifting mechanism.
What are the characteristics of stable air? * Good visibility, steady precipitation, stratus clouds. Poor visibility, steady precipitation, stratus clouds. Poor visibility, intermittent precipitation, cumulus clouds.
What is a characteristic of stable air? * Stratiform clouds. Fair weather cumulus clouds. Temperature decreases rapidly with altitude.
When an air mass is stable, which of these conditions are most likely to exist? * Numerous towering cumulus and cumulonimbus clouds. Moderate to severe turbulence at the lower levels. Smoke, dust, haze, etc., concentrated at the lower levels with resulting poor visibility.
Which is characteristic of stable air? * Cumuliform clouds. Excellent visibility. Restricted visibility.
Which is a characteristic typical of a stable air mass? * Cumuliform clouds. Showery precipitation. Continuous precipitation.
Which are characteristics of a cold air mass moving over a warm surface? * Cumuliform clouds, turbulence, and poor visibility. Cumuliform clouds, turbulence, and good visibility. Stratiform clouds, smooth air, and poor visibility.
Which is true regarding a cold from occlusion? The air ahead of the warm front * Is colder than the air behind the overtaking cold front. Is warmer than the air behind the overtaking cold front. Has the same temperature as the air behind the overtaking cold front.
The conditions most favorable to wave formation over mountainous areas are a layer of * Stable air at mountain top altitude and a wind of at least 20 knots blowing across the ridge. Unstable air at mountain top altitude and a wind of at least 20 knots blowing across the ridge. Moist, unstable air at mountain top altitude and a wind of less than 5 knots blowing across the ridge.
One of the most dangerous features of mountain waves is the turbulent areas in and * Below rotor clouds. Above rotor clouds. Below lenticular clouds.
Virga is best described as * Streamers of precipitation trailing beneath clouds which evaporates before reaching the ground. Wall cloud torrents trailing beneath cumulonimbus clouds which dissipate before reaching the ground. Turbulent areas beneath cumulonimbus clouds.
If airborne radar is indicating an extremely intense thunderstorm echo, this thunderstorm should be avoided by a distance of at least * 20 miles. 10 miles. 5 miles.
What visible signs indicate extreme turbulence in thunderstorm? * Base of the clouds near the surface, heavy rain, and hail. Low ceiling and visibility, hail, and precipitation static. Cumulonimbus clouds, very frequent lightning, and roll clouds.
Which weather phenomenon signals the beginning of the mature stage of a thunderstorm? * The start of rain. The appearance of an anvil top. Growth rate of clouds is maximum.
What feature is normally associated with the cumulus stage of thunderstorm? * Roll cloud. Continuous updraft. Beginning of rain at the surface.
During the life cycle of a thunderstorm, which stage is characterized predominately by downdrafts? * Mature. Developing. Dissipating.
The most severe weather conditions, such as destructive winds, heavy hail, and tornadoes, are generally associated with * Slow-moving warm fronts which slope above the tropopause. Squall lines. Fast-moving occluded fronts.
What minimum distance should exist between intense radar echoes before any attempt is made to fly between these thunderstorms? * 20 miles. 30 miles. 40 miles.
During an IFR cross-country flight you picked up rime icing which you estimate is 1/2 “ thick on the leading edge of the wings. You are now below the clouds al 2000 feet AGL and are approaching your destination airport under VFR. Visibility under the clouds is more than 10 miles, winds at the destination airport are 8 knots right down the runway, and the surface temperature is 3 degrees Celsius. You decide to: * Use a faster than normal approach and landing speed. Approach and land at your normal speed since the ice is not thick enough to have any noticeable effect. Fly your approach slower than normal to lessen the “wind chill” effect and break up the ice.
Frost covering the upper surface of an airplane wing usually will cause. * The airplane to stall at an angle of attack that is higher than normal. The airplane to stall at an angle of attack that is lower than normal. Drag factors so large that sufficient speed cannot be obtained for takeoff.
A characteristic of the stratosphere is * An overall decrease of temperature with an increase in altitude. A relatively even base altitude of approximately 35.000 feet. Relatively small changes in temperature with an increase in altitude.
Which feature is associated with the tropopause? * Absence of wind and turbulent conditions. Absolute upper limit of cloud formation. Abrupt change in temperature lapse rate.
A jet stream is defined as wind of* 30 knots or greater. 40 knots or greater. 50 knots or greater.
The primary cause of all changes in the Earth’s weather is * Variation of solar energy received by the Earth’s regions. Changes in air pressure over the Earth’s surface. Movement of the air masses.
If the air temperature is +8ºC at an elevation of 1.350 feet and a standard (average) temperature lapse rate exists, what will be the approximate freezing level? * 3.350 feet MSL. 5.350 feet MSL. 9.350 feet MSL.
A common type of ground or surface based temperature inversion is that which is produced * Warm air being lifted rapidly aloft in the vicinity of mountainous terrain. The movement of colder air over warm air, or the movement f warm air under cold air. Ground radiation on clear, cool nights when the wind is light.
What feature is associated with temperature inversion? * A stable layer of air. An unstable layer of air. Air mass thunderstorms.
Which weather conditions should be expected beneath a low-level temperature inversion layer when the relative humidity is high?* Smooth air and poor visibility due to fog, haze, or low clouds. Light wind shear and poor visibility due t haze and light rain. Turbulence air and poor visibility due to fog, low stratus-type clouds, and showery precipitation.
Which force, in the Northern Hemisphere, acts at a right angle to the wind and deflects it to the right until parallel to the isobars? * Centrifugal. Pressure gradient. Coriolis.
Clouds, fog, or dew will always form when * Water vapor condenses. Water vapor is present. The temperature and dew point are equal.
To which meteorological condition does the term “dew point” refer? * The temperature to which air must be cooled to become saturated. The temperature at which condensation and evaporation are equal. The temperature at which dew will always form.
The amount of water vapor which air can hold largely depends on * Relative humidity. Air temperature. Stability of air.
What enhances the growth rate of precipitation? * Advective action. Upward currents. Cyclonic movement.
The formation of either predominantly stratiform or predominantly cumuliform clouds is dependent upon the * Source of lift Stability of air being lifted Temperature of the air being lifted.
Stability can be determined from which measurement of the atmosphere? * Low-level winds. Ambient lapse rate. Atmospheric pressure.
What determines the structure or type of clouds which form as a result of air being forced to ascend? * The method by which the air is lifted. The stability of the air before lifting occurs. The amount of condensation nuclei present after lifting occurs.
What type of clouds will be formed if very stable moist air is forced up slope? * First stratified clouds and then vertical clouds. Vertical clouds with increasing height. Stratified clouds with little vertical development.
What type clouds can be expected when an unstable air mass is forced to ascend a mountain slope? * Layered clouds with little vertical development. Stratified clouds with considerable associated turbulence. Clouds with extensive vertical development.
Which of the following combinations of weather producing variables would likely result in cumuliform-type clouds, good visibility, rain showers, and possible clear-type icing in clouds? * Unstable, moist air, and no lifting mechanism. Stable, dry air, and orographic lifting. Unstable, moist air, and orographic lifting.
The suffix “nimbus”, used in naming clouds, means a * Cloud with extensive vertical development. Rain cloud. Dark massive, towering cloud.
What are the four families of clouds * Stratus, cumulus, nimbus, and cirrus. Clouds formed by updrafts, fronts, cooling layers of air, and precipitation into warm air. High, middle, low, and those with extensive vertical development.
A high cloud is composed mostly of * Ozone. Condensation nuclei. Ice crystals.
Which family of clouds is least likely to contribute to structurl icing on an aircraft? * Low clouds. High clouds. Clouds with extensive vertical development.
Which clouds have the greatest turbulence? * Towering cumulus. Cumulonimbus. Altocumulus castellanus.
Standing lenticular clouds, in mountainous areas, indicate * An inversion. Unstable air. Turbulence.
The presence of standing lenticular altocumulus clouds is a good indication of * A jet stream. Very strong turbulence. Heavy icing conditions.
Ice pellets encountred during flight normally are evidence that *+ A warm front has passed. A warm front is about to pass. there are thunderstroms in the area.
An air mass is a body of air that * Has similar cloud formations associated with it. Creates a wind shift as it moves across the Earth’s surface. Covers an extensive area and has fairly uniform properties of temperature and moisture.
Which weather phenomenon is always associated with the passage of a frontal system? * A wind change. An abrupt decrease in pressure. Clouds, either ahead or behind the front.
If you fly into severe turbulence, which flight condition should you attempt to maintain? * Constant airspeed (VA). Level flight attitude. Constant altitude and constant airspeed.
If severe turbulence is encountered during your IFR flight, the airplane should be slowed to the design maneuvering speed because the * Maneuverability of the airplane will be increased. Amount of excess load than can be imposed on the wing will be decreased. Airplane will stall at a lower angle of attack, giving an increased margin of safety.
A pilot reporting turbulence that momentarily causes slight, erratic changes in altitude and/or attitude should report it as * Light turbulence. Moderate turbulence. Light chop.
What are the requirements for the formation of a thunderstorm? * A cumulus cloud with sufficient moisture. A cumulus cloud with sufficient moisture and an inverted lapse rate. Sufficient moisture, an unstable lapse rate, and a lifting action.
Which weather phenomenon signals the beginning of the mature stage of a thunderstorm? * The start of rain at the surface. Growth rate of clouds is maximum. Strong turbulence in the cloud.
During the life cycle of a thunderstorm, which stage is characterized predominately by downdrafts? * Cumulus. Dissipating. Mature.
What is an indication that downdrafts have developed and the thunderstorm cell has entered the mature stage? * The anvil top has completed its development. Precipitation begins to fall from the cloud base. A gust front forms.
Where do squall lines most often develop? * In an occluded front. In a cold air mass. Ahead of a cold front.
Which thunderstorm generally produce the most severe conditions, such as heavy hail and destructive? * Warm front. Squall line. Air mass.
What is indicated by the term “embedded thunderstorms”? * Severe thunderstorms are embedded within a squall line. Thunderstorms are predicted to develop in a stable air mass. Thunderstorms are obscured by massive cloud layers and cannot be seen.
Which weather phenomenon is always associated with a thunderstorm? * Lightning. Heavy rain showers. Supercooled raindrops.
Which procedure is recommended if a pilot should unintentionally penetrate embedded thunderstorm activity? * Reverse aircraft heading or proceed toward an area of known VFR conditions. Reduce airspeed to maneuvering speed and maintain a constant altitude. Set power for recommended turbulence penetration airspeed and attempt to maintain a level fligh tattitude.
Which conditions result in the formation of frost? * The temperature of the collecting surface is at or below freezing and small droplets of moisture are failing. When dew forms and the temperature is below freezing. Temperature of the collecting surface is below the dew point of surrounding air and the dew point is colder than freezing.
Why is frost considered hazardous to flight operation? * Frost changes the basic aerodynamic shape of the airfoil. Frost decreases control effectiveness. Frost causes early airflow separation resulting in a loss of lift.
In which meteorological environment is aircraft structural icing most likely to have the highest rate of accumulation? * Cumulonimbus clouds. High humidity and freezing temperature. Freezing rain.
Where does wind shear occur? * Exclusively in thunderstorms. Wherever there is an abrupt decrease in pressure and/or temperature. With either a wind shift or a wind speed gradient at any level in the atmosphere.
What is an important characteristic of wind shear? * It is primarily associated with the lateral vortices generated by thunderstorms. It usually exists only in the vicinity of thunderstorms, but may be found near a strong temperature inversion. It may be associated with either a wind speed gradient at any level in the atmosphere.
Which is a characteristic of low-level wind shear as it relates to frontal activity? * With a warm front, the most critical period is before the front passes the airport. With a cold front, the most critical period is just before the front passes the airport. Turbulence will always exist in wind-shear conditions.
Which forecast provides specific information concerning expected sky cover, cloud tops, visibility, weather, and obstructions to vision in a route formal? * Area Forecast. Terminal Forecast. Transcribed Weather Broadcast.
The Surface Analysis Chart depicts * Frontal locations and expected movement, pressure centers, cloud coverage, and obstructions to vision at the time of chart transmission. Actual frontal positions, pressure patterns, temperature, dew point, wind, weather, and obstructions at the valid time of the chart. Actual pressure distribution, frontal systems, cloud heights and coverage, temperature, dewpoint, and wind at the time shown on the chart.
Which provides a graphic display of both VFR and IFR weather? * Surface Weather Map. Radar Summary Chart. Weather Depiction Chart.
When total sky cover is few or scattered, the height shown on the Weather Depiction Chart is the * Top of the lowest layer. Base of the lowest layer. Base of the highest layer.
Which weather chart depicts conditions forecast to exist at a specific time in the future? * Freezing Level Chart. Weather Depiction Chart. 12-Hour Significant Weather Prognostication Chart.
What is the upper limit of the low level Significant Weather Prognostic Chart? * 30.000 feet. 24.000 feet. 18.000 feet.
The most current en route and destination weather information for an instrument flight should be obtained from the * AFSS. ATIS broadcast. Notices to Airman Publications.
What is meant by the Special METAR weather observation for KBOI? SPECI KBOI 09185AZ 32005KT 1 1/2SM RA BR OVC007 17/16 A2990 RMK RAB12 * Rain and fog obscuring two-tenths of the sky, rain began at 1912z. Rain and mist obstructing visibility, rain began at 1812Z. Rain and overcast at 1200 feet AGL.
To best determine observed weather conditions between weather reporting stations, the pilot should refer to * Pilot reports. Area Forecast. Prognostic charts.
A pilot reporting turbulence that momentarily causes slight, erratic changes in altitude and/or attitude should report it as. * Light chop. Light turbulence. Moderate turbulence.
When turbulence causes changes in altitude and/or attitude, but aircraft control remains positive, that should be reported as * Light. Severe. Moderate.
Turbulence that is encountered above 15.000 feet AGL not associated with cumuliform cloudiness, including thunderstorms, should be reported as * Severe turbulence. Clear air turbulence. Convective turbulence.
What values are used for Winds Aloft Forecasts? * True directions and MPH. True directions and Knots. Magnetic direction and knots.
SIGMETs are issued as a warning of weather conditions which are hazardous * To all aircraft. Particularly to heavy aircraft. Particularly to light airplanes.
Which correctly describes the purpose of convective SIGMETs (WST)? * They consist of an hourly observation of tornadoes, significant thunderstorm activity,and large hail stone activity. They contain both an observation and a forecast of all thunderstorm and hailstone activity. The forecast is valid for 1 hour only. They consist of either an observation and a forecast or just a forecast for tornadoes, significant thunderstorm activity, or hail greater than or equal 3/4 inch in diameter.
What type of in flight Weather Advisories provides an en route pilot with information regarding the possibility of moderate icing, moderate turbulence, winds of 30 knots or more at the surface and extensive mountain obscurement? * Convective SIGMETs and SIGMETs. Severe Weather Forecast Alerts (AWW) AND SIGMETs. AIRMETs and Center Weather Advisories (CWA).
What single reference contains information regarding volcanic eruption, that is occurring or expected to occur? * In-flight Weather Advisories. Terminal Area Forecast (TAF). Weather Depiction Chart.
The Hazardous In flight Weather Advisory Service (HIWAS) is a broadcast service over service over selected VORs that provides * SIGMETs and AIRMETs at 15 minutes and 45 minutes past the hour for the first hour after issuance. Continuous broadcast of in flight weather advisories. SIGMETs, CONVECTIVE SIGMETs and AIRMETs at 15 minutes and 45 minutes past the hour.
When are severe weather watch bulletins (AWW) issued? * Every 12 hours as required. Every 24 hours as required. Unscheduled and issued as required.
SIGMETs are issued as a warning of weather conditions potentially hazardous * Particularly to light aircraft. To all aircraft. Only to light aircraft operations.
Which meteorological condition is issued in the form of a SIGMET (WS)? * Widespread sand or dust storms lowering visibility to less than 3 miles. Moderate icing. Sustained winds of 30 knots or greater at the surface.
The surface Analysis Chart depicts * Actual pressure systems, frontal locations, cloud tops, and precipitation at the time shown on the chart. Frontal locations and expected movement, pressure centers, cloud coverage, and obstructions to vision at the time of chart transmission. Actual frontal positions, pressure patterns, temperature, dewpoint, wind, weather, and obstructions to vision at the valid time of the chart.
What important information is provided by the Radar Summary Chart that is not shown on other weather charts? * Lines and cells of hazardous thunderstorms. Types of precipitation. Areas of cloud cover and icing levels within the clouds.
What is the lowest cloud in the stationary group associated with mountain wave? * Rotor Cloud Standing Lenticular Low Stratus.
At which location does Coriolis force have the least effect on wind direction * At the poles Middle latitudes (30°-60°) At the Equator.
El codigo SKC se utiliza en el METAR cuando? * La Visibilidad es menos a 10 KM. Y hay nubosidad por debajo de 5000 FT. La visibilidad horizontal es mayor a 10 KM. Y el cielo esta despejado. La visbilidad horizontal es menor de 10 KM. Y el viento esta despejado.
En un mapa meteorológico y en una carta de vuelo, un frente oculido se representa por? * Una linea de color púrpura, triángulos y semicirculos colocados sobre éste. Una linea azul y triángulos alternos colocados a lo largo de éste. Una línea de color rojo y semicirculos colocados a lo largo de éste.
En el tropico el mayor peligro para una aeronave en vuelo cerca o dentro de una CB es? * Los rayos y el granizo. La turbulencia. La mala visibilidad.
What conclusion(s) can be drawn from a 500 millibar Constant Pressure Chart for a planned flight at FL 180? * Winds aloft at FL 180 generally flow across the height contours. Observed temperature, wind, and temperature/dew point spread along the proposed route can be approximated. Upper highs, lows, troughs, and ridges will be depicted by the use of lines of equal pressure.
(Refer to Figure 4) What is the height of the tropopause over Kentucky? FL390. FL300 sloping to FL 400 feet MSL. FL340.
Which meteorological conditions are depicted by prognostic chart? * Conditions existing at the time of the observation. Interpretation of weather conditions for geographical areas between reporting stations. Conditions forecast to exist at a specific time shown on the chart.
(Refer to figure 5, SFC-PROG.) The chart symbols shown in the Gulf of Mexico at 12Z and extending into AL, GA, SC and northern FL indicate a * Tropical storm. Hurricane. Tornado originating in the Gulf of Mexico.
What flight planning information can a pilot derive from constant pressure charts? * Clear air turbulence and icing conditions. Levels of widespread cloud coverage. Winds and temperatures aloft.
(Refer to figure 4.) What is the maximum wind velocity forecast in the jet stream shown on the high level Significant Weather Prognostic Chart over Canada? * 80. 103. 130.
A prognostic chart depicts the conditions * Existing at the surface during the past 6 hours. Which presently exist exist from the 1.000-milibar through the 700-milibar level. Forecast to exist at a specific time in the future.
(Refer to figure 6.) What is the meaning of the symbol depicted as used on the U.S. Low-Level Significant Weather Prog Chart? * Showery precipitation (e.g.rainshowers) embedded in an area of continuous rain covering half or more of the area. Continuous precipitation (e.g. rain showers) covering half or more of the area. Showery precipitation (e.g. thunderstorms/rain showers) covering half or more of the area.
(Refer to figure 5, SFC-400MB.) The U.S. Low Level Significant Weather Surface Prog Chart at 00Z indicates that northwestern Colorado and eastern Utah can expect * Moderate or greater turbulence from the surface to FL 240. Moderate or greater turbulence above FL 240. No turbulence is indicated.
(Refer to figure 7.) What weather conditions are depicted within the area indicated by arrow E? * Occasional cumulonimbus,1/8 to 4/8 coverage, bases below 24.000 feet MSL, and tops at 40.000 feet MSL. Frequent embedded thunderstorms, less than 1/8 coverage, and tops at FL370. Frequent lighting in thunderstorms at FL370.
(Refer to figure 7.) What weather conditions are depicted within the area indicated by arrow D? * Forecast isolated thunderstorm, tops at FL 440, more than 1/8 coverage. Existing isolated cumulonimbus clouds, tops above 43.000 feet with less than 1/8 coverage. Forecast isolated embedded cumulonimbus clouds with tops at 43.000 feet MSL, and less than 1/8 coverage.
(Refer to figure 7.) What weather conditions are predicted within the area indicated by arrow C? * Light turbulence at FL 370 within the area outlined by dashes. Moderate turbulence at 32.000 feet MSL. Moderate to sever CAT has been reported at FL 320.
(Refer to figure 7.) What information in indicated by arrow A? * The height of the tropopause in meters above sea level. The height of the existing layer of CAT. The height of the tropopause in hundreds of feet above MSL.
(Refer to figure 7.) What weather conditions are depicted within the area indicated by arrow F? * 1/8 to 4/8 coverage, occasional embedded thunderstorms, maximum tops at 51.000 feet MSL. Occasionally embedded cumulonimbus, bases below 24.000 feet with tops to 48.000 feet. 2/8 to 6/8 coverage, occasional embedded thunderstorms, tops at FL 540.
A ceiling is defined as the height of the * Highest layer of clouds or obscuring phenomena aloft that covers over 6/10 of the sky. Lowest layer of clouds that contributed to the overall overcast. Lowest layer of clouds or obscuring phenomena aloft that is reported as broken or overcast.
What significant sky condition is reported in this METAR observation? METAR KBNA 1250Z 33018KT 290V360 1/2SM R31/2700FT +SNBLSNFG VV0008 00/M03 A2991RMK RERAE42SNB42 * Runway 31 ceiling is 2700 feet. Sky is obscured with vertical visibility of 800 feet. Measured ceiling is 300 feet overcast.
The body of a Terminal Aerodrome Forecast (TAF) covers a geographical proximity within a * 5 nautical mile radius of the center of an airport. 5 statute mile radius from the center of an airport runway complex. 5 to 10 statute mile radius from the center of an airport runway complex.
Which primary source should be used to obtain forecast weather information at your destination for the planned ETA? * Area Forecast. Radar Summary and Weather Depiction Charts. Terminal Aerodrome Forecast (TAF).
A “VRB” wind entry in a Terminal Aerodrome Forecast (TAF) will be indicated when the wind is * 3 knots or less. 6 knots or less. 9 knots or less.
When the visibility is greater than 6 SM on a TAF it is expressed as * 6PSM. P6SM. 6SMP.
What is the forecast wind at 1800z in the following TAF? KMEM 091740Z 1818 00000KT 1/2SM RAFG OVC005= * Calm. Unknown. Not recorded.
From which primary source should you obtain information regarding the weather expected to exist at your destination at your estimated time of arrival? * Weather Depiction Chart. Radar Summary and Weather Depiction Chart. Terminal Aerodrome Forecast.
Which is true regarding the use of airborne weather-avoidance radar for the recognition of certain weather conditions? * The radarscope provides no assurance of avoiding instrument weather conditions. The avoidance of hail is assured when flying between and just clear of the most intense echoes. The clear area between intense echoes indicates that visual sighting of storms can be maintained when flying between the echoes.
When is the wind-group at one of the forecast altitudes omitted at a specific location or station in the Winds and Temperature ALOFT Forecast (FD)? When the wind * Is less 5 knots. Is less 10 knots. At the altitude is within 1.500 feet of the station elevation.
When are severe weather watch bulletins (WW) issued? * Every 12 hours as required. Every 24 hours as required. Unscheduled and issued as required.
SIGMETs are issued as a warning of weather conditions potentially hazardous. * particularly to light aircraft. to all aircraft. only to light aircraft operations.
A pilot planning to depart at 1100z on an IFR flight is particularly concerned about the hazard of icing. What sources reflect the most accurate information on icing conditions (current and forecast) at the time of departure? * Low-level Significant Weather Prognostic Chart, and the Area Forecast. The Area Forecast, and the Freezing Level Chart. Pilot weather reports (PIREPs), AIRMETs, and SIGMETs.
Cual es la cantidad de octas de nubes cuando se reporta BKN? * De 3 a 5. De 5 a 7. De 1 a 2.
Una expresión “Aproximación Visual”: * La puede solicitar el piloto o ser iniciada por el controlador La ordena el controlador Cancelo plan instrumentos, continuo plan visual.
Una autorización de salida en plan IFR debe contener en su orden: *+ Limite, Vía, Mantenga y Restricciones Vía, Mantenga, Limite y Restricciones Restricciones, Limite, Mantenga, Vía.
La respuesta radar de un respondedor en MODO 3/A código 2000 es: * Vuelo militar en misión especial Vuelo VFR en Colombia Solicito asignación de código.
Una aeronave llegando a Cali con FL240 y estima el VOR de CLO a las 22:20, recibe la siguente autorización: "Autorizado al VOR de CLO, descienda y mantenga 15,000 FT, hora prevista de aproximación 22:35", al tener una falla de comunicación…. * InIciar el descenso sobre el VOR de CLO a las 22:35 desde FL240 siempre y cuando haya colacionado la autorización. Inciar el descenso sobre el VOR de CLO a las 22:35 desde 15000 FT siempre y cuando haya colacionado la autorización. Inciar el descenso sobre el VOR de CLO a las 22:20 desde FL240 siempre y cuando haya colacionado la autorización.
En la fraseología normalizada, en la autorización a la espera en un fijo no debe faltar: * tiempo de alejamiento y EAT Tiempo de alejamiento y ETA Nombre de radioayuda y EAT.
Si estamos volando en espacio aéreo Clase A, esperamos: * Autorizaciones para IFR Autorizaciones IFR e información VFR Información de aeronaves en VFR.
En el contenido de una autorización IFR debemos colacionar: * Solo los SID´s, niveles y restricciones Con decir RECIBIDO es suficiente Toda la autorización.
Bajo que situación un piloto puede cambiar su plan de vuelo IFR a VFR * Cuando lo solicite el controlador Si esta volando en espacio aéreo Clase A Si esta volando en espacio aéreo Clase D.
De una aeronave secuestrada con flaps abajo después de aterrizar significa: * Déjenme solo, no intervengan La situación esta desesperada Pueden subir, situación controlada.
Durante una aproximación por referencia visual, se espera que: *+ El piloto asume la navegación pero la separación VFR/IFR la hace ATS El control ceda su responsabilidad al piloto por ser vuelo VFR El piloto es responsable de la separación con otros tránsitos IFR y VFR.
what designed airspace associated with an airport becomes inactive when the tower at that airport is not in operation? * Class D, which then becomes Class C. Class D, which then becomes Class E. Class B.
En el espacio aéreo CTR se: * Organiza el tránsito en el círculo de aeródromo Establecen secuencias de aproximación por instrumentos Provee separación de aeronaves en ruta.
La máxima velocidad de una aeronave categoría A en un circulo de espera a 12.000 es:(OACI) * 170 Kts 230 kts 240 kts.
En una aproximación IFR, se llama FAF al punto donde una aeronave: * Inicia la aproximación intermedia Esta alineada con la pista para aterrizar Efectúa la espera en fijo primario.
Si un vuelo IFR nocturno se realiza en una FIR, será un vuelo: *+ NO controlado Controlado como todo IFR Controlado si las condiciones son IMC.
Si una aeronave militar intercepta una aeronave civil en vuelo y le hace la siguiente señal: Baja tren de aterrizaje y mantiene luces de aterrizaje encendidos, esto significa: * Sígame Prosiga Aterrice aquí.
Un piloto esta en la obligación de notificar a los servicios ATS respecto a la hora estimada a un punto de notificación, cuando exista una diferencia del que aparece en el plan de vuelo actualizado de (minutos): * 1 2 3.
Una aeronave aproximando a Bogota con 17000 FT y estimando el VOR de BOG a las 09:32 UTC, recibe la siguente autorización: "Autorizado al VOR de BOG, descienda y mantenga 13000 FT, hora prevista de aproximación 09:55", luego sufre falla de comunicación * Inciar el descenso sobre el VOR de BOG a las 09:55 desde 13000 FT siempre y cuando haya colacionado la autorización y esté IMC. Iniciar descenso y aproximación tan pronto como se le presente la falla. Aterrizar en el aeródromo adecuado mas cercano si esta en condiciones VMC.
Vat is defined as speed at threshold based on 1.3 times stall speed in the landing configuration as: * Maximum certificated landing mass Minimum certificated landing mass Maximum certificated takeoff mass.
La prioridad máxima de aterrizaje para aeronaves es? * Aeronaves VIP1. Aeronaves ambulancia. Aeronaves en emergencia.
What is the maximum indicated airspeed allowed in the airspace underlyning Class B airspace? * 156 Knots. 200 Knots. 230 Knots.
Straight departure is one in which the initial departure track is within: * 5° of the alignment of the runway center line 15° of the alignment of the runway center line 25° of the alignment of the runway center line.
For IFR operations off established airways, ROUTE OF LIGHT portion of an IFR plane should list VOR navigational aids which are no more than * 40 miles apart 70 miles apart 80 miles apart.
The initial approach segments, commences at… and ends at… * IF and FAF IAF and IF IF and OM.
Flight procedures for racetrack and reversal procedures are based on average achieved bank angle of: * 30° Bank angle 25° Bank angle, or the bank angle giving a rate of turn of 3°/second, wichever is less. 45° bank angle.
While maintaining a constant heading, a relative bearing of 15° doubles in 6 minutes. The time to the station being used is * 3 minutes 6 minutes 12 minutes.
The optimum Glide Path angle of an ILS is: * 4.0° 3.0° 1.5°.
The maximum aircraft operations holding speed (kts) above 14.000 ft to 20.000 ft, according OACI: * 230 240 265.
Each required flight crewmembers is required to keep his or her shoulders harness fastened * during takeoff and landing only when passengers are aboard the aircraft while the crewmembers are at their station unless he or she is unable to perform requires duties during takeoff and landing unless he or she is unable to perform required duties.
En caso de una bomba abordo que se requiera de una acción inmediata se deberá utilizar el código:* Bravo Romeo Bravo Wiskey Bravo Tango.
In class A airspace, the following flights are permitted:* VFR only IFR only Special VFR only.
The difference between MDA and DA is:* MDA is for Precision approach and DA is for non Precision approach MDA is Minimum departure altitude and DA is decision altitude MDA is for non precision approach and DA is for precision approach.
For the purposes of wake turbulence separation, ATC classifies aircraft as: * Heavy, large and small A, B, C,D Number of engines.
The MINIMUM altitude to intercept the glide slope path on a precision approach is: *+ Minimum altitude leaving VOR in final approach Glide Slope intercept altitude DA value for this procedure.
The difference among a PUBLISHED ROUTE and UNPLUBLISHED ROUTE is: * PUBLISHED route include minimum en-route altitude and the other do not PUBLISHED is for IFR flights only and UNPUBLISHED is for VFR flights only PUBLISHED is authorized route and UNPUBLISHED is non authorized route.
EXPECTED APPROACH TIME means the time: * The flight is estimating arrive at the primary navigation aid The flight is clear for approach In case of communication failure, it is the time at which an approach must begin.
RADAR CONTACT is defined as: * The situation which exists when the radar position of a particular aircraft is seen on a radar display. The situation which exists when the radar position of a particular aircraft is seen and identified on a radar display. None of the above.
Clearance limit is: * The point at which as aircraft is expected to reduce airspeed The point until the aircraft is expected to maintain visual reference and keeps clear of clouds The point at which an aircraft is granted an air traffic control clearance.
THRESHOLD is defined as: * The beginning of that portion of the runway usable for landing The beginning of that portion of the runway used for warm-up. The beginning of that portion of the takeoff runway.
If an airplane is consuming 95 pounds of fuel per hour at a cruising altitude of 6500 feet and the groundspeed is 173 knots , how much fuel is required to travle 450 NN? *+ 112 kg 265 pounds 284 pounds.
En una aproximación ADF la altura mínima de descenso es de: * 500 pies 450 pies 350 pies.
Los pilotos de transporte de línea y copilotos con licencia comercial de aviones deben realizar una serie de proeficiencias y repasos al año para cumplir con los requisitos de la UAEAC, en cuanto a simuladores de vuelo se refiere, el número de estos es de: * Dos veces al año Tres veces al año Cada 2 años.
Como se interpreta si las luces PAPI se observan desde su avion totalmente blancas: * Voy en trayectoria . Voy ligeramente alto. Voy muy alto.
La UTA es un espacio aéreo: * Categoria A, con vuelos IFR y VMC. Categoria A restringido a instrumentos. Categoria B únicamente.
Cúal es la cantidad máxima de horas anuales que un piloto con licencia PCA, que vuela para una empresa de servicio aereo regular y una aeronave de mas de 5700 KGS puede volar? * 500 Horas 700 Horas 900 Horas.
Se define como área de maniobras en un aeródromo a: * La utilizada o definida como área de vuelo para operación de aeronaves sujeta a restricciones o limitaciones. Todo tipo de aérea de movimiento en la cual se desplace una aeronave. Parte del aeródromo que se utiliza para despegues, aterrizajes y rodaje de aeronaves excluyendo las plataformas.
Se define como aerovía: * El área controlada por la torre de control El área comprendida entre dos aeropuertos conformada por 2 radio ayudas y con un ancho de 12 Millas náuticas. El área de control dispuesta en forma de corredor y equipada con ayudas para la navegación.
Se considera combustible básico para el despacho de una aeronave: * El suficiente para volar del aeropuerto de origen a destino. El suficiente para volar de origen a destino más el alterno. El suficiente para volar de origen a destino más extra.
Se considera combustible de reserva para el despacho de una aeronave: * El suficiente para volar y aterrizar en el aeropuerto alterno más lejano incluido en el plan de vuelo. El suficiente para sostener por 30 minutos. El suficiente para sostener por 45 minutos.
Se considera combustible mínimo para el despacho de una aeronave: * El suficiente para llegar de origen a destino. El suficiente para llegar de origen a destino mas reserva. La suma de combustible básico + reserva + contingencia(si aplica) + sostenimiento.
El RAC comprende: * 9 partes 11 partes 24 partes.
El RAC se define como: * Reglamento del aire de Colombia. Reglamento Aeronáutico de Colombia Reglamento de Aviones Colombia.
La parte cuarta del RAC corresponde a: * Aeródromos e instalaciones Personal Aeronáutico Normas de aeronavegabilidad y Operación de aeronaves.
La parte Séptima del RAC corresponde a: * Personal Aeronáutico Normas de Aeronavegabilidad y Operación de Aeronaves Régimen Sancionatorio.
Para poder servir como copiloto de una empresa de transporte aéreo regular, este debe de haber cumplido con un chequeo de rutas dentro de un período de: * 12 meses 18 meses 24 meses.
La palabra Notam se define como: * Notice to the Aircraft. Notice to the Airman Notice to the Air maintenance operator.
El Límite de horas mensuales para un tripulante de aeronaves en Colombia según el RAC es de: * 80 horas 90 horas 85 horas.
Una calle de rodaje de baja velocidad forma ángulo de: * 120° con respecto el eje longitudinal de la pista. 90° con respecto al eje longitudinal de la pista 90° con respecto a la torre de control.
El control de horas mensuales y anuales de un tripulante de vuelo es responsabilidad de: * El operador. El tripulante La Aeronáutica Civil.
El tiempo máximo de servicio de un tripulante de cabina de mando para tripulación sencilla de aeronaves de transporte aéreo regular es de: * 12 horas 12:30 horas 14 horas.
Espacio aéreo controlado visual por TWR: * ATZ CTR TMA.
El RAC permite libre acceso a la cabina de mando de una aeronave comercial a: * Un controlador de tráfico aéreo Un inspector de la UAEAC Un inspector de la Fuerza Aérea Colombiana.
La responsabilidad Final en la operación de una aeronave depende de: * La UAEAC El piloto al mando La empresa explotadora.
El RAC exige que por encima de ___________ se debe llevar suficiente oxígeno para suministro de todos los miembros de la tripulación en caso de una despresurización. * 11.000 pies 10.000 pies 12.000 pies.
De acuerdo al RAC, para despachar una aeronave cuando el aeropuerto de origen esta por debajo de los mínimos de aterrizaje se requiere tener: * Un aeropuerto alterno de despegue ubicado a 30 minutos de vuelo a velocidad normal de crucero con un motor inoperativo. Un aeropuerto alterno de despegue ubicado a 45 minutos de vuelo a velocidad normal de crucero con un motor inoperativo. Un aeropuerto alterno de despegue ubicado a 1 hora de vuelo a velocidad normal de crucero con un motor inoperativo.
La separación vertical mínima entre dos aeronaves operando en espacio aéreo RVSM es de: * 500 pies 1.000 pies 2.000 pies.
Se define en una aproximación de precisión como DH (Decision Height) a: * La altura mínima en la cual una aeronave debe iniciar una aproximación frustrada en caso de no tener contacto visual. La altura mínima a la que puede descender una aeronave con el piloto automático enganchado. Un punto geográfico exacto,que coincide con una altura especifica en la cual una aeronave de no tener contacto visual debe iniciar una aproximación frustrada.
De acuerdo al RAC, se pierde autonomía para actuar como piloto al mando o copiloto de una aeronave si en los_____ días precedentes no se han efectuado _____ despegues y aterrizajes. * 60 y 3 90 y 3 60 y 2.
Durante una aproximación por referencia visual, se espera que:..* El control ceda su responsabilidad al piloto por ser un vuelo VFR El piloto sea responsable de la separación con otros tránsitos IFR y VFR El piloto asuma la navegación pero la separación VFR/IFR la efectué el ATS.
Una aeronave vuela dentro de un FIR con: * Altitudes Alturas Niveles de vuelo.
En una aproximación IFR, se denomina FAF (Final Approach Fix) al punto donde una * Inicia la aproximación intermedia Inicia la maniobra de aproximación frustrada Se encuentra alineada con la pista para aterrizar.
Si un vuelo IFR nocturno se realiza en un FIR, será un vuelo:* NO controlado Controlado si las condiciones son IMC NO controlado si las condiciones son VMC.
La legislación Aérea Colombiana se encuentra publicada en:* Manual de Reglamentos Aeronáuticos Código de comercio AIP de Colombia.
Si una aeronave militar intercepta una aeronave civil en vuelo y le hace la siguiente señal: Baja tren de aterrizaje y mantiene luces de aterrizaje encendidos, esto significa:* sígame Aterrice aquí Usted esta en peligro. Sígame.
Un piloto esta en la obligación de notificar a los servicios ATS respecto a la hora estimada a un punto de notificación, cuando exista una diferencia de la que aparece en el plan de vuelo actualizado de:* 1 minuto 2 minutos 3 minutos.
La expresión aeronave OSCAR PAPA significa una aeronave en vuelo de: Operación VIP presidencial* Ambulancia Orden público.
De acuerdo al RAC, el piloto al mando de una aeronave podrá desviarse de una autorización emitida por el ATC sí:* Esta pone en riesgo la seguridad del vuelo. Una situación de emergencia es declarada a y b son correctas.
Cuando una aeronave se encuentre en una situación de emergencia que ponga en peligro la seguridad de las personas o de las aeronaves y se requiera de tomar medidas que infrinjan los reglamentos o procedimientos locales, el piloto al mando deberá pasar un informe escrito dentro de los siguientes:* 5 días 8 días 10 días.
Para recobro de autonomía como Piloto Comercial de aviones después de un receso de 90 días, el tripulante deberá efectuar_____ despegues y aterrizajes ante un instructor calificado, en el equipo que desea recobrar autonomía. * 4 3 5.
El cumplimiento de las normas aeronáuticas en Colombia es principal función de: * El ministerio de transporte La UAEAC La secretaría de transporte Aéreo de Colombia.
Para actuar como tripulante efectivo de una aeronave en Colombia, se deberá tener en posesión siempre: * Licencia Expedida por la Aeronáutica Civil de Colombia y Certificado Medico Certificado de Carencia de Estupefacientes y Certificado de antecedentes disciplinarios a y b son correctas.
Cuando una aeronave se aproxime de frente con otra, las dos variaran su rumbo para evitar una colisión de acuerdo a los siguientes parámetros: * Ambas aeronaves viraran a la derecha Ambas aeronaves viraran a la izquierda La aeronave de mayor envergadura iniciara un ascenso y por consecuente la otra un descenso.
De acuerdo a lo establecido en el RAC, excepto cuando sea necesario para el despegue o aterrizaje de una aeronave o cuando lo autorice la autoridad competente, no se efectuaran vuelos IFR a un nivel inferior a la altitud mínima establecida de: * 2.000 pies por encima del obstáculo mas alto en un radio de 5 millas en terreno montañoso. 1.000 pies por encima del obstáculo mas alto en un radio de 5 millas en terreno no montañoso. a y b son correctos.
Las condiciones mínimas de alcance visual en Colombia por encima de 10,000 pies serán de* 8 km 5 millas 5 km 3 millas 8 millas 5 kms.
Cada vez que el piloto al mando de una aeronave ejerza la autoridad de emergencia, deberá reportar a la UAEAC en los siguientes:* 5 días hábiles 8 días hábiles 10 días hábiles.
La responsabilidad de reportar irregularidades mecánicas que ocurran durante el tiempo de vuelo es de:* El piloto al mando El técnico de mantenimiento El explotador de la aeronave.
La responsabilidad de que se efectúe un prevuelo antes de iniciar un vuelo en una aeronave es de:* El piloto al mando El copiloto El técnico de mantenimiento.
El reglamento del aire es parte de:* El Manual de Operaciones de Vuelo de un operador El Reglamento Aeronáutico de Colombia El Manual de Reglamentos Aeronáuticos.
Se define como hora prevista de aproximación * Hora en la que el ATC prevé que una aeronave que llega después de experimentar una demora, abandone el fijo de espera para completar su aproximación, a fin de aterrizar La hora en la cual se estima que una aeronave aterrice en su aeropuerto de destino Ninguna de las anteriores.
La altura mínima de decisión para una aproximación ILS categoría I es igual:* 100 pies 200 pies 250 pies.
Se considera como aproximación de precisión: * ILS VOR DME Contacto radar.
Si volamos en condiciones VMC queremos decir que:* Estamos en condiciones de vuelo visual y volando IMC Estamos en condiciones de vuelo visual y nuestro plan de vuelo puede ser IFR Todas las respuestas anteriores son ciertas.
If on an IFR flight plan, you are advised to be in radar contact, you must:* Obey all instructions from ATC regardless Obey all instructions from ATC only while in cloud Follow all possible instructions and advise ATC when unable to comply.
When flying a commercial route you are familiar with and you are in congested airspace, you may instead of reading back all clearances and instructions repetitively simply:* Say “Roger” to ATC without repeating your call sign or flight number again, once you know ATC recognizes your voice Simply click your mike when answering obvious things to ATC. So as to not congest the frequency Read back all clearances including your call sign and or flight # even if this is going to congest the frequency even more.
You must advice ATC if you cannot climb or descent to your assigned altitude if you cannot maintain at least: * No restriction if cleared at pilot discretion 500 ft on climb and 1.500 ft on descent 500 FPM.
An IFR clearance must include the following: * Dep. Airport, aircraft ID. Flight level and route Only the aircraft ID and transponder code if advised as filed on a passenger carrying Aircraft ID, Clearance limit, Dep. Procedure, route and transponder code.
You are expected to read back clearances containing altitude assignments, radar vectors or any instructions requiring verification: * True False As well as those containing wind direction and intensity.
If while on a hold and this is not your IAF, you lose 2 way voice communications, you should:* Leave the hold at the EFC time Immediately fly in VMC and land as soon as possible Stay on the hold until your EAT.
You filed from SKBO to SKMD W23-ABL-W36-FELIX-W25-RNG. Just before ABL you experience 2 way Com. Failure and just before this you were told to fly ABL Direct RNG. With regards to the correct route to fly you should: * Maintain present course and filed altitude until Com is reestablished Continue your flight route as per original flight plan as it was read back on ground to clearance delivery before takeoff. As you cross ABL, fly direct RNG.
During preflight of a transport category aircraft that undergoes a progressive maintenance program the ELT should: * Tested and when done, ensure the switch is selected to ARMED Tested by selecting 243 mhz on the radio and listening for a tone when the switch is selected to ON Selected to ARMED and if absolutely necessary to test, it should only be done by selecting it to ON for no more than 3 seconds during ONLY the first 5 minutes of every hour.
Señalar lo que es cierto * El MMEL (Master MEL) lo prepara el explotador El MEL es más restrictivo que el MMEL (Master MEL) El MEL es preparado por el explotador del avión.
Which report should be made to ATC without a specific request when not in radar contact? * Entering instrument meteorological conditions. When leaving final approach fix inbound on final approach. Correcting an E.T.A. any time a previous E.T.A. is in error in excess of 2 minutes.
For which speed variation should you notify ATC? * When the ground speed changes more than 5 knots. When the average true airspeed changes 5 percent or 10 knots, whichever is greater. Any time the ground speed changes 10 MPH.
What does declaring “minimum fuel” to ATC imply?* Traffic priority is needed to the destination airport. Emergency handling is required to the nearest useable airport. Merely an advisory that indicates an emergency situation is possible should any undue delay occur.
When ATC has not imposed any climb or descent restrictions and aircraft are within 1,000 feet of assigned altitude, pilots should attempt to both climb and descend at a rate of between * 500 feet per minute and 1,000 feet per minute. 500 feet per minute and 1,500 feet per minute 1,000 feet per minute and 2,000 feet per minute.
For IFR planning purposes, what are the compulsory reporting points when using VOR/DME or VORTAC fixes to define a direct route not on established airways? * Fixes selected to define the route. There are no compulsory reporting points unless advised by ATC. At the change over points.
What action should you take if your No. 1 VOR receiver malfunctions while operating in controlled airspace under IFR? Your aircraft is equipped with two VOR receivers. The No. 1 receiver has VOR/Localizer/Glide Slope capability, and the No. 2 has only VOR/Localizer capability. * Report the malfunction immediately to ATC. Continue the flight as cleared, no report is required. Continue the approach and request a VOR or NDB approach.
During an IFR flight in IMC, a distress condition is encountered, (fire, mechanical, or structural failure). The pilot should Not hesitate to declare an emergency and obtain an amended clearance.* Wait until the situation is immediately perilous before declaring an emergency. Contact ATC and advise that an urgency condition exists and request priority consideration.
During the en route phase of an IFR flight, the pilot is advised “Radar service terminated.” What action is appropriate? * Set transponder to code 1200. Resume normal position reporting Activate the IDENT feature of the transponder to re-establish radar contact.
During an IFR flight in IMC, you enter a holding pattern (at a fix that is not the same as the approach fix) with an EFC time of 1530. At 1520 you experience complete two-way communications failure. Which procedure should you follow to execute the approach to a landing? * Depart the holding fix to arrive at the approach fix as close as possible to the EFC timeandcompletethe approach. Depart the holding fix at the EFC time, and complete the approach. Depart the holding fix at the earliest of the flight planned ETA or the EFC time, and complete the approach.
Which procedure should you follow if you experience two-way communications failure while holding at a holding fix with an EFC time? (The holding fix is not the same as the approach fix.) * Depart the holding fix to arrive at the approach fix as close as possible to the EFC time. Depart the holding fix at the EFC time. Proceed immediately to the approach fix and hold until EFC.
Which procedure should you follow if, during an IFR flight in VFR conditions, you have two-way radio communications failure? * Continue the flight under VFR and land as soon as practicable. Continue the flight at assigned altitude and route, start approach at your ETA, or, if late, start approach upon arrival. Land at the nearest airport that has VFR conditions.
What altitude and route should be used if you are flying in IMC and have two-way radio communications failure? * Continue on the route specified in your clearance,fly at an altitude that is the highest of last assigned altitude, altitude ATC has informed you to expect, or the MEA. Fly direct to an area that has been forecast to have VFR conditions, fly at an altitude that is at least 1,000 feet above the highest obstacles along the route. Descend to MEA and , if clear of clouds, proceed to the nearest appropriate airport. If not clear of clouds, maintain the highest of the MEA’s along the clearance route.
In the event of two way radio communications failure while operating on an IFR clearance in VFR conditions the pilot should continue * By the route assigned in the last ATC clearance received. The flight under VFR and land as soon as practical. The flight by the most direct route to the fix specified in the last clearance.
While flying on an IFR flight plan, you experience two-way communications radio failure while in VFR conditions. In this situation, you should continue your flight under * VFR and land as soon as practicable. VFR and proceed to your flight plan destination. IFR and maintain the last assigned route and altitude to your flight plan destination.
What is the significance of an ATC clearance which reads “... CRUISE SIX THOUSAND ...”?* The pilot must maintain 6,000 feet until reaching the IAF serving the destination airport, then execute the published approach procedure. Climbs may be made to, or descents made from, 6,000 feet at the pilot’s discretion. The pilot may utilize any altitude from the MEA/MOCA to6,000feet, but each change in altitude must be reported to ATC.
What is the recommended climb procedure when a nonradar departure control instructs a pilot to climb to the assigned altitude? * Maintain a continuous optimum climb until reaching assigned altitude and report passing each 1,000 foot level. Climb at a maximum angle of climb to within 1,000 feet of the assigned altitude, then 500 feet per minute the last 1,000 feet. Maintain an optimum climb on the center line of the airway without intermediate level offs until 1,000 feet below assigned altitude, then 500 to 1500 feet per minute.
When departing from an airport not served by a control tower, the issuance of a clearance containing a void time indicates that * ATC will assume the pilot has not departed if no transmission is received before the void time. The pilot must advise ATC as soon as possible, but no later than 30 minutes, of their intentions if not off by the void time. ATC will protect the airspace only to the void time.
What response is expected when ATC issues an IFR clearance to pilots of airborne aircraft? * Read back the entire clearance as required by regulation. Read back those parts containing altitude assignments or vectors and any part requiring verification. Read-back should be unsolicited and spontaneous to confirm that the pilot understands all instructions.
Which clearance items are always given in an abbreviated IFR departure clearance? (Assume radar environment.) * Altitude, destination airport, and one or more fixes which identify the initial route of flight. Destination airport, altitude, DP Name, Number, and/or Transition, if appropriate. Clearance limit, DP Name, Number, and/or Transition, if appropriate.
On the runup pad, you receive the following clearance from ground control: CLEARED TO THE DALLAS-LOVE AIRPORT AS FILED--MAINTAIN SIX THOUSAND--SQUAWK ZERO SEVEN ZERO FOUR JUST BEFORE DEPARTURE--DEPARTURE CONTROL WILL BE ONE TWO FOUR POINT NINER. An abbreviated clearance, such as this, will always contain the * Departure control frequency. Destination airport and route. Requested enroute altitude.
When may ATC request a detailed report of an emergency even though a rule has not been violated? * When priority has been given. Any time an emergency occurs. When the emergency occurs in controlled airspace.
Which clearance procedures may be issued by ATC without prior pilot request? * DPs, STARs, and contact approaches. Contact and visual approaches. DPs, STARs, and visual approaches.
What is the significance of an ATC clearance which reads “...CRUISE SIX THOUSAND...? * The pilot must maintain 6,000 until reaching the IAF serving the destination airport, then execute thepublished approach procedure. It authorizes a pilot to conduct flight at any altitude from minimum IFR altitude up to and including 6,000. The pilot is authorized to conduct flight at any altitude from minimum IFR altitude up to and including 6,000, but each change in altitude must be reported to ATC.
A “CRUISE FOUR THOUSAND FEET” clearance would mean that the pilot is authorized to* Vacate 4,000 feet without notifying ATC. Climb to, but not descend from 4,000 feet, without further ATC clearance. Use any altitude from minimum IFR to 4,000 feet, but must report leaving each altitude.
While on an IFR flight, a pilot has an emergency which causes a deviation from an ATC clearance. What action must be taken? * Notify ATC of the deviation as soon as possible. Squawk 7700 for the duration of the emergency. Submit a detailed report to the chief of the ATC facility within 48 hours.
An abbreviated departure clearance “...CLEARED AS FILED...” will always contain the name* And number of the STAR to be flown when filed in the flight plan. Of the destination airport filed in the flight plan. Of the first compulsory reporting point if not in a radar environment.
14 CFR part 1 defines VLE as * Maximum landing gear extended speed. Maximum landing gear operating speed. Maximum leading edge flaps extended speed.
14 CFR part 1 defines VNE as * Maximum landing gear extended speed. Never-exceed speed. Maximum nose wheel extend speed.
14 CFR part 1 defines VY as * Speed for best rate of descent Speed for best angle of climb Speed for best rate of climb.
Which is the correct symbol for the stalling speed or the minimum steady flight speed at which the airplane is controllable? * VS VS1 VSO.
Define Calibrated Air Speed? * Normal operating speed. Is the indicated airspeed corrected for instrument errors, position error Speed corrected by Warm climate.
14 CFR part 1 defines VF as * Design flap speed. Flap operating speed. Maximum flap extended speed.
Which is the correct symbol for the stalling speed or the minimum steady flight speed in a specified configuration? * VS VS1. VSO.
Which airspeed would a pilot be unable to identify by the color coding of an airspeed indicator? * The never-exceed speed. The power-off stall speed. The maneuvering speed.
If a standard rate turn is maintained, how long would it take to turn 360°? * 1 minute. 2 minutes. 3 minutes.
To determine pressure altitude prior to takeoff, the altimeter should be set to * The current altimeter setting. 29.92 inches Hg and the altimeter indication noted. The field elevation and the pressure reading in the altimeter setting window noted.
What is an operational difference between the turn coordinator and the turn-and-slip indicator? The turn coordinator * Is always electric, the turn-and-slip indicator is always vacuum-driven. Indicates bank angle only, the turn-and-slip indicator indicates rate of turn and coordination. Indicates roll rate, rate of turn, and coordination, the turn-and-slip indicator indicates rate of turn and coordination.
What is an advantage of an electric turn coordinator if the airplane has a vacuum system for other gyroscopic instruments? * It is a backup in case of vacuum system failure. It is more reliable than the vacuum-driven indicators. It will not tumble as will vacuum-driven turn indicators.
A Standard rate turn is : * A turn at a rate of 3% min at 100 Kts (TAS in Kts / 3º) + 5 A turn at a rate of 3º / sec.
Magnetic dip is responsible for: * Northerly turning errors only in the northern hemisphere counteracted by magnetic dip Nothing on a commercial transport category Aircraft equipped with EFIS Errors during acceleration or turns on only magnetic compasses.
Pitot static instruments are connected as follows on a high Performance aircraft *+ Airspeed = pitot only, Altimeter = static only Airspeed = pitot + static, Altimeter = static only, Vertical speed = pitot + alternate Static Airspeed = pitot + static ,Altimeter = static only, Vertical speed = static only.
At higher elevation airports the indicated airspeed: * Is the same as the TAS Is higher and so is the resultant ground speed Remains the same but the ground speed is faster than at lower altitude airports.
TAS is the actual speed your airplane moves through undisturbed air. As density altitude increases your TAS: *+ Should remain the same regardless Should increase about 2 Kts / 1.000 Ft increment Should increase for a given TAS corrected for instrument and installation error.
Pressure altitude is: * Displayed when the altimeter setting is corrected for temperature at sea level Displayed when the altimeter setting is set on the QNH window Displayed when 29.92 is set on the altimeter regardless of outside temp. + pressure.
Density altitude is used for:* Only for flying altitudes below 18.000 Ft. Calculating TAS with 29.92 set on the Alt. Performance calculations with OAT charts.
While on the ground with the correct altimeter setting your altimeter should read:* Within 100 Ft. for CAT I ILS approved airplanes Within 20 Ft. of the other altimeter Within 75 Ft. of the actual field elevation.
If on a takeoff roll below 80 Kts. one of the airspeed indicators is under reading by exactly 30 Kts., you should: * Continue the take off and report to Maintenance at next stop as you still have 1 reliable ASI available Continue the takeoff and consult the MEL before writing out a Maintenance report Reject the take off, return to the gate and report to Maintenance assuming all the delay consequences.
Compass correction cards * Are not required on EFIS equipped aircraft Are not required on transport category aircraft Must be current and installed on all certified aircraft.
Pitch instruments are: * Attitude, vertical speed and altimeter Altimeter, airspeed and VSI Attitude indicator, altimeter, airspeed and VSI.
When performing a VOR accuracy check at a VOR test point on an airfield, the VOR can be considered serviceable if it is within _______ of the published radial: * 2º to either side No deviations are allowed for IFR 4º to either side.
If a VOR check is desired and no VOT is available, the check: * Must be made by maintenance personnel with special equipment Is not necessary for aircraft certified for CAT II ILS approaches Can be made between 2 VOR´S within the aircraft and the error must not exceed 4º.
What information does a Mach meter present? * The ratio of aircraft true airspeed to the speed of sound. The ratio of aircraft indicated airspeed to the speed of sound. The ratio of aircraft equivalent airspeed, corrected for installation error, to the speed of sound.
Under what condition is pressure altitude and density altitude the same value? * At standard temperature. When the altimeter setting is 29.92" Hg. When indicated, and pressure altitudes are the same value on the altimeter.
Under which condition will pressure altitude be equal to true altitude? * When the atmospheric pressure is 29.92" Hg. When standard atmospheric conditions exist. When indicated altitude is equal to the pressure altitude.
Which condition would cause the altimeter to indicate a lower altitude than actually flown (true altitude)? *+ Air temperature lower than standard. Atmospheric pressure lower than standard. Air temperature warmer than standard.
Under what condition will true altitude be lower than indicated altitude with an altimeter setting of 29.92" Hg? * In warmer than standard air temperature. In colder than standard air temperature. When density altitude is higher than indicated altitude.
Altimeter setting is the value to which the scale of the pressure altimeter is set so the altimeter indicates * True altitude at field elevation. Pressure altitude at field elevation. Pressure altitude at sea level.
How can you obtain the pressure altitude on flights below 18,000 feet? * Set your altimeter to 29.92" Hg. Use your computer to change the indicated altitude to pressure altitude. Contact an FSS and ask for the pressure altitude.
How can you determine the pressure altitude on an airport without a tower or FSS? * Set the altimeter to 29.92" Hg and read the altitude indicated. Set the altimeter to the current altimeter setting of a station within 100 miles and correct this indicated altitude with local temperature Use your computer and correct the field elevation for temperature.
Which altitude is indicated when the altimeter is set to 29.92" Hg? * Density. Pressure. Standard.
At an altitude of 6,500 feet MSL, the current altimeter setting is 30.42" Hg. The pressure altitude would be approximately * 7,500 feet. 6,000 feet. 6,500 feet.
The pressure altitude at a given location is indicated on the altimeter after the altimeter is set to * The field elevation. 29.92" Hg. The current altimeter setting.
If the outside air temperature increases during a flight at constant power and at a constant indicated altitude, the true airspeed will Decrease and true altitude will increase. Increase and true altitude will decrease. Increase and true altitude will increase.
Altimeter setting is the value to which the scale of the pressure altimeter is set so the altimeter indicates * Pressure altitude at sea level. True altitude at field elevation. Pressure altitude at field elevation.
Pressure altitude is the altitude read on your altimeter when the instrument is adjusted to indicate height above * Sea level. The standard datum plane. Ground level.
When an altimeter is changed from 30.11" Hg to 29.96" Hg, in which direction will the indicated altitude change and by what value? * Altimeter will indicate 15 feet lower. Altimeter will indicate 150 feet lower. Altimeter will indicate 150 feet higher.
En route at FL290, the altimeter is set correctly, but not reset to the local altimeter setting of 30.57" Hg during descent. If the field elevation is 650 feet and the altimeter is functioning properly, what is the approximate indication upon landing? * 715 feet. 1,300 feet. Sea level.
En route at FL290, your altimeter is set correctly, but you fail to reset it to the local altimeter setting of 30.26" Hg during descent. If the field elevation is 134 feet and your altimeter is functioning properly, what will it indicate after landing? * 100 feet MSL. 474 feet MSL. 206 feet below MSL.
Which of the following defines the type of altitude used when maintaining FL210? * Indicated. Pressure. Calibrated.
What is the procedure for setting the altimeter when assigned an IFR altitude of 18,000 feet or higher on a direct flight off airways? * Set the altimeter to 29.92" Hg before takeoff. Set the altimeter to the current altimeter setting until reaching the assigned altitude, then set to 29.92" Hg. Set the altimeter to the current reported setting for climb out and 29.92" Hg upon reaching 18,000 feet.
While you are flying at FL250, you hear ATC give an altimeter setting of 28.92" Hg in your area. At what pressure altitude are you flying? * 24,000 feet. 25,000 feet. 26,000 feet.
If you are departing from an airport where you cannot obtain an altimeter setting, you should set your altimeter * On 29.92" Hg. On the current airport barometric pressure, if known. To the airport elevation.
The local altimeter setting should be used by all pilots in a particular area, primarily to provide for* The cancellation of altimeter error due to nonstandard temperatures aloft. Better vertical separation of aircraft. More accurate terrain clearance in mountainous areas.
How should you preflight check the altimeter prior to an IFR flight? * Set the altimeter to 29.92" Hg. With current temperature and the altimeter indication, determine the true altitude to compare with the field elevation. Set the altimeter first with 29.92" Hg and then the current altimeter setting. The change in altitude should correspond to the change in setting. Set the altimeter to the current altimeter setting.The indication should be within 75 feet of the actual elevation for acceptable accuracy.
One characteristic that a properly functioning gyro depends upon for operation is the * Ability to resist precession 90° to any applied force. Resistance to deflection of the spinning wheel or disc. Deflecting force developed from the angular velocity of the spinning wheel.
If a 180° steep turn is made to the right and the aircraft is rolled out to straight-and-level flight by visual references, the attitude indicator * Should immediately show straight-and-level flight. Will show a slight skid and climb to the right. May show a slight climb and turn.
As a rule of thumb, altitude corrections of less than 100 feet should be corrected by using a* Full bar width on the attitude indicator. Half bar width on the attitude indicator. Two bar width on the attitude indicator.
Which condition during taxi is an indication that an attitude indicator is unreliable? * The horizon bar tilts more than 5° while making taxi turns. The horizon bar vibrates during warmup. The horizon bar does not align itself with the miniature airplane after warmup.
What pretakeoff check should be made of the attitude indicator in preparation for an IFR flight? * The horizon bar does not vibrate during warmup. The miniature airplane should erect and become stable within 5 minutes. The horizon bar should erect and become stable within 5 minutes.
During normal coordinated turns, what error due to precession should you observe when rolling out to straight-and-level flight from a 180° steep turn to the right? * A straight-and-level coordinated flight indication. The miniature aircraft would show a slight turn indication to the left. The miniature aircraft would show a slight descent and wings-level attitude.
Errors in both pitch and bank indication on an attitude indicator are usually at a maximum as the aircraft rolls out of a * 180° turn. 270° turn. 360° turn.
When an aircraft is accelerated, some attitude indicators will precess and incorrectly indicate a * Climb. Descent. Right turn.
When an aircraft is decelerated, some attitude indicators will precess and incorrectly indicate a * Lefft turn. Climb. Descent.
While cruising at 160 knots, you wish to establish a climb at 130 knots. When entering the climb (full panel), it is proper to make the initial pitch change by increasing back elevator pressure until the * Attitude indicator, airspeed, and vertical speed indicate a climb. Vertical speed indication reaches the predetermined rate of climb. Attitude indicator shows the approximate pitch attitude appropriate for the 130-knot climb.
While cruising at 190 knots, you wish to establish a climb at 160 knots. When entering the climb (full panel), it would be proper to make the initial pitch change by increasing back elevator pressure until the * Attitude indicator shows the approximate pitch attitude appropriate for the 160-knot climb. Attitude indicator, airspeed, and vertical speed indicate a climb. Airspeed indication reaches 160 knots.
Prior to starting an engine, you should check the turn – and – slip indicator to determine if the Needle indication properly corresponds to the angle of the wings or rotors with the horizon. Needle is approximately centered and the tube is full of fluid. Ball will move freely from one end of the tube of other when the aircraft is rocked.
What indication should be observed on a turn coordinator during a left turn while taxiing? * The miniature aircraft will show a turn to the left and the ball remains centered. The miniature aircraft will show a turn to the left and the ball moves to the right. Both the miniature aircraft and the ball will remain centered.
What indications should you observe on the turn-and-slip indicator during taxi? * The ball moves freely opposite the turn, and the needle deflects in the direction of the turn. The needle deflects in the direction of the turn, but the ball remains centered. The ball deflects opposite the turn, but the needle remains centered.
What does the miniature aircraft of the turn coordinator directly display? * Rate of roll and rate of turn. Angle of bank and rate of turn. Angle of bank.
What indications are displayed by the miniature aircraft of a turn coordinator? * Rate of roll and rate of turn. Direct indication of bank angle and pitch attitude. Indirect indication of bank angle and pitch attitude.
What indication is presented by the miniature aircraft of the turn coordinator? * Indirect indication of the bank attitude. Direct indication of the bank attitude and the quality of the turn. Quality of the turn.
Which instrument indicates the quality of a turn? * Attitude indicator Heading indicator or magnetic compass. Ball of the turn coordinator.
What force causes an airplane to turn? * Rudder pressure or force around the vertical axis. Vertical lift component. Horizontal lift component.
What force causes an airplane to turn?.* Rudder pressure or force around the vertical axis. Vertical lift component. Horizontal lift component.
The rate of turn at any airspeed is dependent upon * The horizontal lift component. The vertical lift component. Centrifugal force.
When airspeed is decreased in a turn, what must be done to maintain level flight? * Decrease the angle of bank and/or increase the angle of attack. Increase the angle of bank and/or decrease the angle of attack. Increase the angle of attack.
During a skidding turn to the right, what is the relationship between the component of lift, centrifugal force, and load factor? * Centrifugal force is less than horizontal lift and the load factor is increased. Centrifugal force is grater than horizontal lift and the load factor is increased. Centrifugal force and horizontal lift are equal and the load factor is decreased.
What is the relationship between centrifugal force and the horizontal lift component in a coordinated turn? * Horizontal lift exceeds centrifugal force. Horizontal lift and centrifugal force are equal. Centrifugal force exceeds horizontal lift.
When airspeed is increased in a turn, what must be done to maintain a constant altitude? * Decrease the angle of bank Increase the angle of bank and/or decrease the angle of attack Decrease the angle of attack.
During standard-rate turns, which instrument is considered primary for bank? * Heading indicator. Turn and slip indicator or turn coordinator. Attitude indicator.
a half-standard rate turn is maintained, how long would it take to turn 360°? * 1 minute. 2 minutes. 4 minutes.
If a standard rate turn is maintained, how long would it take to turn 180°? * 1 minute 2 minutes 3 minutes.
If a half-standard rate turn is maintained, how much time would be required to turn clockwise from a heading of 090° to a heading of 180°? * 30 seconds 1 minute 1 minute 3 seconds.
During a constant-bank level turn, what effect would an increase in airspeed have on the rate and radius of turn? * Rate of turn would increase, and radius of turn would increase. Rate of turn would decrease, and radius of turn would decrease. Rate of turn would decrease, and radius of turn would increase.
If a standard rate turn is maintained, how much time would be required to turn to the right from a heading of 090° to a heading of 270°? * 1 minute. 2 minutes. 3 minutes.
If a standard rate turn is maintained, how much time would be required to turn to the left from a heading of 090° to a heading of 300°? * 30 seconds. 40 seconds. 50 seconds.
If a half-standard rate turn is maintained, how long would it take to turn 135°? * 1 minute. 1 minute 20 seconds. 1 minute 30 seconds.
Rate of turn can be increased and radius of turn decreased by * Decreasing airspeed and shallowing the bank. Decreasing airspeed and increasing the bank. Increasing airspeed and increasing the bank.
The displacement of a turn coordinator during a coordinated turn will * Indicate the angle of bank. Remain constant for a given bank regardless of airspeed. Increase as angle of bank increases.
(Refer to Figure 8.) What changes in control displacement should be made so that “2” would result in a coordinated standard rate turn? * Increase left rudder and increase rate of turn. Increase left rudder and decrease rate of turn. Decrease left rudder and decrease angle of bank.
(Refer to Figure 8.) Which illustration indicates a coordinated turn? * 3 1 2.
(Refer to Figure 8.) Which illustration indicates a skidding turn? * 2 1 3.
(Refer to Figure 8.) What changes in control displacement should be made so that “1” would result in a coordinated standard rate turn? * Increase right rudder and decrease rate of turn. Increase right rudder and increase rate of turn. Decrease right rudder and increase angle of bank.
(Refer to Figure 8.) Which illustration indicates a slipping turn? * 1 3 2.
What pretakeoff check should be made of a vacuum-driven heading indicator in preparation for an IFR flight? * After 5 minutes , set the indicator to the magnetic heading of the aircraft and check for proper alignment after taxi turns. After 5 minutes, check that the heading indicator card aligns itself with the magnetic heading of the aircraft. Determine that the heading indicator does not precess more than 2° in 5 minutes of ground operation.
On the taxi check, the magnetic compass should * Swing opposite to the direction of turn when turning from north. Exhibit the same number of degrees of dip as the latitude. Swing freely and indicate known headings.
What should be the indication on the magnetic compass as you roll into a standard rate turn to the left from an east heading in the Northern Hemisphere? * The compass will initially indicate a turn to the right. The compass will remain on east for a short time, then gradually catch up to the magnetic heading of the aircraft. The compass will indicate the approximate correct magnetic heading if the roll into the turn is smooth.
What should be the indication on the magnetic compass as you roll into a standard rate turn to the right from an easterly heading in the Northern Hemisphere? * The compass will initially indicate a turn to the left. The compass will remain on east for a short time, then gradually catch up to the magnetic heading of the aircraft. The compass will indicate the approximate correct magnetic heading if the roll into the turn is smooth.
What should be the indication on the magnetic compass as you roll into a standard rate turn to the right from a south heading in the Northern Hemisphere? * The compass will indicate a turn to the right, but at a faster rate than is actually occurring. The compass will initially indicate a turn to the left. The compass will remain on south for a short time,then gradually catch up to the magnetic heading of the aircraft.
On what headings will the magnetic compass read most accurately during a level 360° turn, with a bank of approximately 15°? * 135° through 225°. 90° and 270°. 180° and 0°.
What causes the northerly turning error in a magnetic compass? * Coriolis force at the mid-latitudes. Centrifugal force acting on the compass card. The magnetic dip characteristic.
What should be the indication on the magnetic compass when you roll into a standard rate turn to the left from a south heading in the Northern Hemisphere? * The compass will indicate a turn to the left, but at a faster rate than is actually occurring. The compass will initially indicate a turn to the right. The compass will remain on south for a short time, then gradually catch up to the magnetic heading of the aircraft.
What should be the indication on the magnetic compass as you roll into a standard rate turn to the right from a westerly heading in the Northern Hemisphere? * The compass will initially show a turn in the opposite direction,then turn to a northerly indication but lagging behind the actual heading of the aircraft. The compass will remain on a westerly heading for a short time, then gradually catch up to the actual heading of the aircraft. The compass will indicate the approximate correct magnetic heading if the roll into the turn is smooth.
What should be the indication on the magnetic compass as you roll into a standard rate turn to the right from a northerly heading in the Northern Hemisphere? * The compass will indicate a turn to the right, but at a faster rate than is actually occurring. The compass will initially indicate a turn to the left. The compass will remain on north for a short time,then gradually catch up to the magnetic heading of the aircraft.
What should be the indication on the magnetic compass as you roll into a standard rate turn to the left from a west heading in the Northern Hemisphere? * The compass will initially indicate a turn to the right. The compass will remain on west for a short time, then gradually catch up to th e magnetic heading of the aircraft. The compass will indicate the approximate correct magnetic heading if the roll into the turn is smooth.
What should be the indication on the magnetic compass as you roll into a standard rate turn to the left from a north heading in the Northern Hemisphere? * The compass will indicate a turn to the left, but at a faster rate than is actually occurring. The compass will initially indicate a turn to the right. The compass will remain on north for a short time,then gradually catch up to the magnetic heading of the aircraft.
Which practical test should be made on the electric gyro instruments prior to starting an engine? * Check that the electrical connections are secure on the back of the instruments. Check that the attitude of the miniature aircraft is wings level before turning on electrical power. Turn on the electrical power and listen for any unusual or irregular mechanical noise.
En route at FL290, your altimeter is set correctly, but you fail to reset it to the local altimeter setting of 30.26" Hg during descent. If the field elevation is 134 feet and your altimeter is functioning properly, what will it indicate after landing?.. * 100 feet MSL. 474 feet MSL. 206 feet below MSL.
If both the ram air input and drain hole of the pitot system are blocked, what airspeed indication can be expected? * No variation of indicated airspeed in level flight even if large power changes are made. Decrease of indicated airspeed during a climb. Constant indicated airspeed during a descent.
If both the ram air input and the drain hole of the pitot system are blocked, what reaction should you observe on the airspeed indicator when power is applied and a climb is initiated out of severe icing conditions? * The indicated airspeed would show a continuous deceleration while climbing. The airspeed would drop to, and remain at, zero. No change until an actual climb rate is established, then indicated airspeed will increase.
What indication should a pilot observe if an airspeed indicator ram air input and drain hole are blocked? * The airspeed indicator will react as an altimeter. The airspeed indicator will show a decrease with an increase in altitude. No airspeed indicator change will occur during climbs or descents.
What would be the indication on the VSI during entry into a 500 FPM actual descent from level flight if the static ports were iced over? * The indication would be in reverse of the actual rate of descent (500 FPM climb). The initial indication would be a climb, then descent at a rate in excess of 500 FPM. The VSI pointer would remain at zero regardless of the actual rate of descent.
If, while in level flight, it becomes necessary to use an alternate source of static pressure vented inside the airplane, which of the following should the pilot expect? * The altimeter and airspeed indicator to become inoperative. The gyroscopic instruments to become inoperative The vertical speed to momentarily show a climb.
During flight, if the pitot tube becomes clogged with ice, which of the following instruments would be affected? * The airspeed indicator only. The airspeed indicator and the altimeter. The airspeed indicator, altimeter, and Vertical Speed Indicator.
If while in level flight, it becomes necessary to use an alternate source of static pressure vented inside the airplane, which of the following variations in instrument indications should the pilot expect? * The vertical speed to momentarily show a descent. The altimeter to read higher than normal. The vertical speed to show a climb.
(Refer to Figure 9.) Identify the system that has failed and determine a corrective action to return the airplane to straight-and-level flight. * Static/pitot system is blocked,lower the nose and level the wings to level-flight attitude by use of attitude indicator. Vacuum system has failed, reduce power, roll left to level wings, and pitchup to reduce airspeed. Electrical system has failed,reduce power,roll left to level wings, and raise the nose to reduce airspeed.
(Refer to Figure 10.) What is the flight attitude? One instrument has malfunctioned. * Climbing turn to the right. Climbing turn to the left. Descending turn to the right.
(Refer to Figure 11.) What is the flight attitude? One system which transmits information to the instruments has malfunctioned. * Level turn to the right. Level turn to the left. Straight-and-level flight.
What is the correct sequence in which to use the three skills used in instrument flying? * Aircraft control, cross-check, and instrument interpretation. Instrument interpretation, cross-check, and aircraft control. Cross-check, instrument interpretation, and aircraft control.
What are the three fundamental skills involved in attitude instrument flying? * Instrument interpretation, trim application, and aircraft control. Cross-check, instrument interpretation, and aircraft control. Cross-check, emphasis, and aircraft control.
What is the third fundamental skill in attitude instrument flying? * Instrument cross-check. Power control. Aircraft control.
What is the first fundamental skill in attitude instrument flying? * Aircraft control. Instrument cross-check. Instrument interpretation.
What effect will a change in wind direction have upon maintaining a 3° glide slope at a constant true airspeed? * When ground speed decreases, rate of descent must increase. When ground speed increases, rate of descent must increase. Rate of descent must be constant to remain on the glide slope.
The rate of descent required to stay on the ILS glide slope Must be increased if the ground speed is decreased. Will remain constant if the indicated airspeed remains constant. Must be decreased if the ground speed is decreased. .
To remain on the ILS glidepath, the rate of descent must be * Decreased if the airspeed is increased. Decreased if the ground speed is increased. Increased if the ground speed is increased.
The rate of descent on the glide slope is dependent upon * True airspeed. Calibrated airspeed. Ground speed.
The glide slope and localizer are centered, but the airspeed is too fast. Which should be adjusted initially? * Pitch and power. Power only. Pitch only.
During a precision radar or ILS approach, the rate of descent required to remain on the glide slope will * Remain the same regardless of ground speed. Increase as the ground speed increases. Decrease as the ground speed increases.
The gyroscopic heading indicator is inoperative. What is the primary bank instrument in unaccelerated straight-and-level flight? * Magnetic compass. Attitude indicator. Miniature aircraft of turn coordinator.
What instruments are considered supporting bank instruments during a straight, stabilized climb at a constant rate? * Attitude indicator and turn coordinator. Heading indicator and attitude indicator. Heading indicator and turn coordinator.
What instruments are primary for pitch, bank, and power, respectively, when transitioning into a constant airspeed climb from straight-and-level flight? * Attitude indicator, heading indicator, and manifold pressure gauge or tachometer. Attitude indicator for both pitch and bank airspeed indicator for power. Vertical speed, attitude indicator, and manifold pressure or tachometer.
What is the primary bank instrument once a standard rate turn is established? * Attitude indicator. Turn coordinator. Heading indicator.
As power is increased to enter a 500 feet per minute rate of climb in straight flight, which instruments are primary for pitch, bank, and power respectively? * Attitude indicator, heading indicator, and manifold pressure gauge or tachometer. VSI, attitude indicator, and airspeed indicator. Airspeed indicator, attitude indicator, and manifold pressure gauge or tachometer.
What is the primary pitch instrument during a stabilized climbing left turn at cruise climb airspeed? * Attitude indicator VSI. Airspeed indicator.
What is the primary pitch instrument when establishing a constant altitude standard rate turn?* Altimeter. VSI. Airspeed indicator.
As a rule of thumb, altitude corrections of less than 100 feet should be corrected by using * Two bar widths on the attitude indicator. Less than a full bar width on the attitude indicator. Less than half bar width on the attitude indicator.
What is the initial primary bank instrument when establishing a level standard rate turn? * Turn coordinator. Heading indicator. Attitude indicator.
What instrument(s) is(are) supporting bank instrument when entering a constant airspeed climb from straight-and-level flight? * Heading indicator. Attitude indicator and turn coordinator. Turn coordinator and heading indicator.
What is the primary bank instrument while transitioning from straight-and-level flight to a standard rate turn to the left? * Attitude indicator. Heading indicator. Turn coordinator (miniature aircraft).
As power is reduced to change airspeed from high to low cruise in level flight, which instruments are primary for pitch, bank, and power, respectively? * Attitude indicator, heading indicator, and manifold pressure gauge or tachometer. Altimeter, attitude indicator, and airspeed indicator. Altimeter, heading indicator, and manifold pressure gauge or tachometer.
Which instrument provides the most pertinent information (primary) for bank control in straight-and-level flight? * Turn-and-slip indicator. Attitude indicator. Heading indicator.
Which instruments are considered primary and supporting for bank, respectively, when establishing a level standard rate turn? * Turn coordinator and attitude indicator. Attitude indicator and turn coordinator Turn coordinator and heading indicator.
Which instruments, in addition to the attitude indicator, are pitch instruments? * Altimeter and airspeed only. Altimeter and VSI only. Altimeter, airspeed indicator, and vertical speed indicator.
Which instrument provides the most pertinent information (primary) for pitch control in straight-and-level flight?* Attitude indicator. Airspeed indicator. Altimeter.
Which instruments are considered to be supporting instruments for pitch during change of airspeed in a level turn? * Airspeed indicator and VSI. Altimeter and attitude indicator. Attitude indicator and VSI.
Which instrument is considered primary for power as the airspeed reaches the desired value during change of airspeed in a level turn? * Airspeed indicator. Attitude indicator. Altimeter.
Which instruments should be used to make a pitch correction when you have deviated from your assigned altitude? * Altimeter and VSI. Manifold pressure gauge and VSI. Attitude indicator, altimeter, and VSI.
Conditions that determine the pitch attitude required to maintain level flight are * Airspeed, air density, wing design, and angle of attack. Flightpath, wind velocity, and angle of attack. Relative wind, pressure altitude, and vertical lift component.
Approximately what percent of the indicated vertical speed should be used to determine the number of feet to lead the level-off from a climb to a specific altitude? * 10 percent. 20 percent. 25 percent .
To level off from a descent to a specific altitude, the pilot should lead the level-off by approximately * 10 percent of the vertical speed. 30 percent of the vertical speed. 50 percent of the vertical speed.
For maintaining level flight at constant thrust, which instrument would be the least appropriate for determining the need for a pitch change? * Altimeter. VSI Attitude indicator.
To enter a constant-airspeed descent from level-cruising flight, and maintain cruising airspeed, the pilot should * First adjust the pitch attitude to a descent using the attitude indicator as a reference, then adjust the power to maintain the cruising airspeed. First reduce power, then adjust the pitch using the attitude indicator as a reference to establish a specific rate on the VSI. Simultaneously reduce power and adjust the pitch using the attitude indicator as a reference to maintain the cruising airspeed.
To level off at an airspeed higher than the descent speed, the addition of power should be made, assuming a 500 FPM rate of descent, at approximately * 50 to 100 feet above the desired altitude. 100 to 150 feet above the desired altitude. 150 to 200 feet above the desired altitude.
To level off from a descent maintaining the descending airspeed, the pilot should lead the desired altitude by approximately * 20 feet. 50 feet. 60 feet.
While recovering from an unusual flight attitude without the aid of the attitude indicator, approximate level pitch attitude is reached when the * Airspeed and altimeter stop their movement and the VSI reverses its trend. Airspeed arrives at cruising speed, the altimeter reverses its trend, and the vertical speed stops its movement. Altimeter and vertical speed reverse their trend and the airspeed stops its movement.
During recoveries from unusual attitudes, level flight is attained the instant * The horizon bar on the attitude indicator is exactly overlapped with the miniature airplane. A zero rate of climb is indicated on the VSI. The altimeter and airspeed needles stop prior to reversing their direction of movement.
(Refer to Figure 12.) What is the correct sequence for recovery from the unusual attitude indicated? * Reduce power, increase back elevator pressure, and level the wings. Reduce power, level the wings, bring pitch attitude to level flight. Level the wings, raise the nose of the aircraft to level flight attitude, and obtain desired airspeed.
(Refer to Figure 13.) Which is the correct sequence for recovery from the unusual attitude indicated? * Level wings, add power, lower nose, descend to original attitude, and heading. Add power, lower nose, level wings, return to original attitude and heading. Stop turn by raising right wing and add power at the same time,lower the nose, and return to original attitude and heading.
If an airplane is in an unusual flight attitude and the attitude indicator has exceeded its limits, which instruments should be relied on to determine pitch attitude before starting recovery? * Turn indicator and VSI. Airspeed and altimeter. VSI and airspeed to detect approaching V(S1) or V(MO).
Which is the correct sequence for recovery from a spiraling, nose-low, increasing airspeed, unusual flight attitude? * Increase pitch attitude, reduce power, and level wings. Reduce power, correct the bank attitude, and raise the nose to a level attitude. Reduce power, raise the nose to level attitude, and correct the bank attitude.
In aircraft equipped with constant-speed propellers and normally-aspirated engines, which procedure should be used to avoid placing undue stress on the engine components? When power is being * Decreased, reduce the RPM before reducing the manifold pressure. Increased, increase the RPM before increasing the manifold pressure. Increased or decreased, the RPM should be adjusted before the manifold pressure.
Which statement best describes the operating principle of a constant-speed propeller? * As throttle setting is changed by the pilot, the prop governor causes pitch angle of the propeller blades to remain unchanged. A high blade angle, or increased pitch, reduces the propeller drag and allows more engine power for takeoffs. The propeller control regulates the engine RPM, and in turn, the propeller RPM.
Fuel/air ratio is the ratio between the * Volume of fuel and volume of air entering the cylinder. Weight of fuel and weight of air entering the cylinder. Weight of fuel and weight of air entering the carburetor.
To establish a climb after takeoff in an aircraft equipped with a constant-speed propeller, the output of the engine is reduced to climb power by decreasing manifold pressure and * Increasing RPM by decreasing propeller blade angle. Decreasing RPM by decreasing propeller blade angle. Decreasing RPM by increasing propeller blade angle.
To develop maximum power and thrust, a constant-speed propeller should be set to a blade angle that will produce a* Large angle of attack and low RPM. Small angle of attack and high RPM. Large angle of attack and high RPM.
For takeoff, the blade angle of a controllable-pitch propeller should be set at a * Small angle of attack and high RPM. Large angle of attack and low RPM. Large angle of attack and high RPM.
The reason for variations in geometric pitch (twisting) along a propeller blade is that it * Permits a relatively constant angle of incidence along its length when in cruising flight. Prevents the portion of the blade near the hub from stalling during cruising flight. Permits a relatively constant angle of attack along its length when in cruising flight.
To establish a climb after takeoff in an aircraft equipped with a constant-speed propeller, the output of the engine is reduced to climb power by decreasing manifold pressure and... * Increasing RPM by decreasing propeller blade angle. Decreasing RPM by decreasing propeller blade angle. Decreasing RPM by increasing propeller blade angle.
A fixed-pitch propeller is designed for best efficiency only at a given combination of * Altitude and RPM. Airspeed and RPM. Airspeed and altitude.
Unless adjusted, the fuel/air mixture becomes richer with an increase in altitude because the amount of fuel * Decreases while the volume of air decreases. Remains constant while the volume of air decreases. Remains constant while the density of air decreases.
A flux value is found in: * Most hydraulic systems Fuel control units and is an internal component not governed from the flight deck Directional Gyro systems and has no moving parts.
The intake section of a jet engine is often considered part of the fuselage. The other four parts which are considered part of the engine are * The turbine, combustion chamber, exhaust and reversers The compressor, combustion chamber, turbine and exhaust The Engine struts, compressor, combustion chamber and exhaust.
Hydraulic systems in modern aircraft are usually employed to * Aid in moving heavy control surfaces or accessories. Aid in lowering the landing gear, moving the nose wheel steering and activating the passenger stairs. Help the pilot lower the landing gear in emergency, activate the Power Transfer Unit (S by. electrical power) and engage the thrust reversers.
Large transport category jet airplanes employ: * Constantly heated leading edges for ground deicing Silver colored deicing boots Heated leading edges for de-icing / anti-icing.
The most fuel efficient type engine for commercial passenger transport is * Turbofan with afterburners Turboprop Jet/scramjet (used on the Concorde).
What type measurement is used to rate power on Jet or Fanjet engines * Pounds of thrust Brake horse power Shaft horsepower (SHP).
Modern aircraft electrical systems normally consist of * Standard 350v 600 Hz AC with step-up transformers Engine generators delivering 115v AC including 400Hz and 24v DC systems 12v AC lead-acid batteries and 24v DC static generators.
Aircraft hydraulic systems use: * Aviation grade engine oil as standard fluid Hydraulic fluid Aircraft grade DOT 4 brake fluid.
Where would a pilot find the type of engine oil a specific aircraft uses * Do not even look, call maintenance In the approved Airplane Operating Manual (AOM) or similar In the Certificate of Airworthiness (Powerplant subsection).
If during a preflight a pilot sees that a tire is showing just a thread of canvass, he should: X Consult with a maintenance technician Consult the limits / tolerances in the approved AFM. Look it up in the Maintenance section of the company’s Operations Specifications.
If at an airport JET A or Jet B type fuels are not available for your aircraft, would you: * Top it up with 100/130 LL Avgas not to exceed a 50% ratio Look in the Flight / Maintenance Manual for alternate fuels Not use any other fuel than JET A or Jet B.
Modern aircraft are usually pressurized in flight by using: * Bleed air tapped off from usually the compressor section of the engine Auxiliary Power Units which are installed for this purpose primarily Pressure controlled bleed air tapped off usually from the last stage turbine section of the engine.
At what Mach range does the subsonic flight range normally occur? * Below .75 Mach. From .75 to 1.20 Mach From 1,20 to 2,50 Mach.
How should thrust reversers be applied to reduce landing distance for turbojet aircraft? *+ Immediatly after ground contact. Immediatly prior to touchdown. After applying maximum wheel braking.
What characterizes a transient compressor stall? * Loud, steady roar accompanied by heavy shuddering. Sudden loss of thrust accompanied by loud whine. Intermittent "bang", as backfires and flow reversal take place.
What indicates that a compressor stall has developed and become steady? * Strong vibrations and loud roar. Occasional loud "bang" and flow reversal. Complete loss of power with severe reduction in airspeed.
In turbine engine jargon, a Hot Start is when: * The outside air temperature is above ISA + 45º C The EGT rises rapidly before the N1/N2N/3 reach sufficient speed and the engine is about to exceed limitations The ITT/EGT rises rapidly and exceeds limitations before sufficient turbine/compressor rotation is obtained.
Turbine engines can deliver reverse thrust by. * Bringing the engine from forward thrust into a momentary stop(stationary) then spooled up into reverse thrust. It is merely accelerated as the exhaust is redirected forward Starting the engine in the opposite direction of normal forward thrust.
What other term is used to classify a “Turboprop Engine”: * Propjet Fanjet TPE (Turbocharged Propeller Engine).
Approximately in what percentage do turboprop engines deliver thrust via the Propeller Vs turbine exhaust: * 100% propeller - 0% turbine exhaust 75% propeller - 25% turbine exhaust 20% propeller - 80% turbine exhaust.
High-bypass ratio turbine engines can be compared in operating principle to * A turboprop engine with 200 or more small propeller blades A regular jet engine with afterburner A simple scramjet engine.
A regular gas turbine type engine’s performance is most affected by: * Ambient pressure and humidity Ambient temperature and pressure Mostly humidity in the air Vs ISA conditions.
If a normally aspirated engine’s power output (such as on a C-150 or PA28) engine is measured in shaft horse power (SHP), a turboprop engine’s power output is measured in: * Lbs of thrust ESHP (equivalent shaft horse power) Also SHP.
What is the prime advantage of a fuel injected engine Vs a carburetor type:* A fuel injector better atomizes the fuel for optimum performance A fuel injector is better because it can be adjusted on wing. A fuel injector is less expensive because it does not need carburetor de-icing.
Turbine engines have a compression ratio of say 20:3 just like normal internal combustion engines do, this ratio on turbine type engines is: * Air pressures at the intake Vs the exhaust stage Air pressures at the combustor stage Vs the turbine stage Air pressures at the turbine Vs the exhaust stage.
Turbine engines have a compression ratio just like internal combustion engines * True False Impossible since turbine engines do not have pistons.
Constant speed propellers operate the same on an internal combustion engine as they do on most turboprop engines because they both * Use a hydraulic oil system to feed an over speed solenoid Use engine oil and a propeller governor to maintain a given RPM set by the pilot Use a propeller governor and turbine/turbocharger air to pneumatically control the engine RPM in flight.
With respect to vortex circulation, which is true? * Helicopters generate downwash turbulence, not vortex circulation. The vortex strength is greatest when the generating aircraft is flying fast. Vortex circulation generated by helicopters in forward flight trail behind in a manner similar to wing tip vortices generated by airplanes.
When landing behind a large aircraft, which procedure should be followed for vortex avoidance? * Stay above its final approach flightpath all the way to touchdown. Stay below and to one side of its final approach flightpath. Stay well below its final approach flightpath and land at least 2,000 feet behind.
To avoid possible wake turbulence from a large jet aircraft that has just landed prior to your takeoff, at which point on the runway should you plan to become airborne? * Past the point where the jet touched down. At the point where the jet touched down, or just prior to this point. Approximately 500 feet prior to the point where the jet touched down.
Which procedure should you follow to avoid wake turbulence if a large jet crosses your course from left to right approximately 1 mile ahead and at your altitude * Make sure you are slightly above the path of the jet. Slow your airspeed to VA and maintain altitude and course. Make sure you are slightly below the path of the jet and perpendicular to the course.
During a takeoff made behind a departing large jet airplane, the pilot can minimize the hazard of wingtip vortices by * Being airborne prior to reaching the jet's flightpath until able to turn clear of its wake. Maintaining extra speed on takeoff and climbout. Extending the takeoff roll and not rotating until well beyond the jet's rotation point.
During an approach, the most important and most easily recognized means of being alerted to possible wind shear is monitoring the * Amount of trim required to relieve control pressures. Heading changes necessary to remain on the runway centerline. Power and vertical velocity required to remain on the proper glidepath.
How can you determine if another aircraft is on a collision course with your aircraft? * The nose of each aircraft is pointed at the same point in space. The other aircraft will always appear to get larger and closer at a rapid rate. There will be no apparent relative motion between your aircraft and the other aircraft.
What altimeter setting is required when operating an aircraft at 18,000 feet MSL? * Current reported altimeter setting of a station along the route. Altimeter setting at the departure or destination airport. 29.92 Inches Hg.
When weather information indicates that abnormally high barometric pressure exists, or will be above _____ inches of mercury, flight operations will not be authorized contrary to the requirements published in NOTAMs. * 30.50 31.00 32.00.
After an ATC clearance has been obtained, a pilot may not deviate from that clearance, unless the pilot * Receives an amended clearance or has an emergency. Is operating VFR on top. Requests an amended clearance.
When planning for an emergency landing at night, on of the primary considerations should include * Turning off all electrical switches to save battery power for the landing. Selecting a landing area close to public access, if possible. Landing without flaps to ensure a nose-high landing attitude at touchdown.
After experiencing a powerplant failure at night, one of the primary considerations should include * Planning the emergency approach and landing to an unlighted portion of an area. Maneuvering to, and landing on a lighted highway or road. Turning off all electrical switches to save battery power for landing.
What are some of the hazardous attitudes dealt with in Aeronautical Decision Making (ADM)? * Risk management, stress management, and risk elements. Poor decision making, situational awareness, and judgment. Antiauthority (don't tell me), impulsivity (do something quickly without thinking), macho (I can do it).
Light beacons producing red flashes indicate * A pilot should remain clear of an airport traffic pattern and continue circling. Obstructions or areas considered hazardous to aerial navigation. End of runway warning at departure end.
(Refer to figure 14.) Which symbol does not directly address runway incursion with other aircraft? * Top red. Middle yellow. Bottom yellow.
(Refer to figure 14.) The red symbol at the top would most likely be found * Upon exiting all runways prior to calling ground control. At an intersection where a roadway may be mistaken as a taxiway. Near the approach end of ILS runways.
(Refer to figure 14.) The pilot generally calls ground control after landing when the aircraft is completely clear of the runway. This is when the aircraft * Passes the red symbol shown at the top of the figure. Is on the dashed-line side of the middle symbol. Is past the solid-line side of the middle symbol.
When a pilot recognizes a hazardous thought, he or she then should correct it by applying the corresponding antidote. Which of the following is the antidote for the ANTIAUTHORITY/DON'T TELL ME hazardous attitude? * It won't happen to me. It could happen to me. Not so fast. Think first. Follow the rules. They are usually right.
The basic drive for a pilot to demonstrate the 'right stuff' can have an adverse effect on safety, by * A total disregard for any alternative course of action. Generating tendencies that lead to practices that are dangerous, often illegal, and that may lead to a mishap. Imposing a realistic assessment of piloting skills under stressful conditions.
Most pilots have fallen prey to dangerous tendencies or behavior problems at some time. Some of these dangerous tendencies or behavior patterns which must be identified and eliminated include: * Deficiencies in instrument skills and knowledge of aircraft systems or limitations. Peer pressure, get-there-itis, loss of positional or situation awareness, and operating without adequate fuel reserves. Performance deficiencies from human factors such as, fatigue, illness or emotional problems.
An early part of the Aeronautical Decision Making (ADM) process involves * Taking a self-assessment hazardous attitude inventory test. Understanding the drive to have the 'right stuff.' Obtaining proper flight instruction and experience during training.
Hazardous attitudes which contribute to poor pilot judgment can be effectively counteracted by * Taking meaningful steps to be more assertive with attitudes. Early recognition of hazardous thoughts. Redirecting that hazardous attitude so that appropriate action can be taken.
What is the first step in neutralizing a hazardous attitude in the ADM process? * Dealing with improper judgment. Recognition of hazardous thoughts. Recognition of invulnerability in the situation.
What does good cockpit stress management begin with? * Knowing what causes stress. Good life stress management. Eliminating life and cockpit stress issues.
The passengers for a charter flight have arrived almost an hour late for a flight that requires a reservation. Which of the following alternatives best illustrates the ANTIAUTHORITY reaction?* Those reservation rules do not apply to this flight. The pilot can't help it that the passengers are late. If the pilot hurries, he or she may still make it on time.
While conducting an operational check of the cabin pressurization system, the pilot discovers that the rate control feature is inoperative. He knows that he can manually control the cabin pressure, so he elects to disregard the discrepancy. Which of the following alternatives best illustrates the INVULNERABILITY reaction?* It's too late to fix it now. He can handle a little problem like this. What is the worst that could happen.
Examples of classic behavioral traps that experienced pilots may fall into are: trying to* Assume additional responsibilities and assert PIC authority. Promote situational awareness and then necessary changes in behavior. Complete a flight as planned, please passengers, meet schedules, and demonstrate the 'right stuff.'.
While on an IFR flight, a pilot emerges from a cloud to find himself within 300 feet of a helicopter. Which of the following alternatives best illustrates the 'MACHO' reaction?* He is not too concerned, everything will be alright. He flies a little closer, just to show him. He quickly turns away and dives, to avoid collision.
To help manage cockpit stress, pilots must* Condition themselves to relax and think rationally when stress appears. Be aware of life stress situations that are similar to those in flying. Avoid situations that will improve their abilities to handle cockpit responsibilities.
A pilot and friends are going to fly to an out-of-town football game. When the passengers arrive, the pilot determines that they will be over the maximum gross weight for takeoff with the existing fuel load. Which of the following alternatives best illustrates the RESIGNATION reaction? * He can't wait around to de-fuel, they have to get there on time. Well, nobody told him about the extra weight. Weight and balance is a formality forced on pilots by the FAA.
Which of the following is the final step of the Decide Model for effective risk management and Aeronautical Decision Making?* Estimate. Eliminate. Evaluate.
Which of the following is the first step of the Decide Model for effective risk management and Aeronautical Decision Making?* Identify. Detect. Evaluate.
The Decide Model is comprised of a 6-step process to provide a pilot a logical way of approaching Aeronautical Decision Making. These steps are:* Detect, estimate, choose, identify, do, and evaluate. Determine, eliminate, choose, identify, detect, and evaluate. Determine, evaluate, choose, identify, do, and eliminate.
Aeronautical Decision Making (ADM) is a* Mental process of analyzing all information in a particular situation and making a timely decision on what action to take. Systematic approach to the mental process used by pilots to consistently determine the best course of action for a given set of circumstances. Decision making process which relies on good judgment to reduce risks associated with each flight.
The Aeronautical Decision Making (ADM) process identifies the steps involved in good decision making. One of these steps includes a pilot* Identifying personal attitudes hazardous to safe flight. Developing the 'right stuff' attitude. Making a rational evaluation of the required actions.
The 'taxiway ending' marker* Identifies area where aircraft are prohibited. Indicates taxiway does not continue. Provides general taxiing direction to taxiway.
On a runway equipped with a precision approach, the touch down zone markings are located:* At the end of the 3.000 Ft. TDZ Depends if it’s a CAT I, II or III ILS runway 500 Ft from the beginning of the runway.
The area before a displaced threshold may be used:* Only in emergencies For taxiing and landing rollout only For taxiing, take off and landing rollout.
A red background sign with white lettering denotes:* Warning positions for Non commercial or VFR aircraft Instructions for military aircraft only An entrance to a runway a critical area or an area prohibited to aircraft.
A runway incursion is: * Not possible at airports with separate tower, ground and ramp controllers A serious offence punishable to non licensed ground vehicle driverS Any occurrence that creates a collision hazard on a runway.
On a PAPI you are on a correct glide path if: * You see 2 white lights next to 2 red lights on the R side of the runway You stay within the standard 3º glide path You see 2 white lights on the outside and 2 red lights close to the left margin of the runway.
Runway center line lights are: * White and green White and spaced every 25 Ft except for the last 500 Ft which are red Red for the last 1.000 Ft of the runway.
Airlines may not operate aircraft in uncontrolled airspace: * True, Passenger carrying flights must always be operated in class A, B or C airspaces False, It is allowed if the airline has foreign registered aircraft operating in Colombia False, it is allowed so long as the OPS SPECS say so.
If you are cleared to a VOR on a descent from Fl. 190 to 9.000 Ft. and your IAS is 279 Kts, when at the VOR: * You must slow down to 200 Kts You are legal since you did not exceed 280 Kts You are illegal unless specifically authorized by ATC.
On an approach to an airport, you have been advised you are in radar contact and are vectored for the approach, the controller asks you fly slightly below the MEA, you should: * Not obey, since you may be in or encounter clouds Obey to all instructions since you were advised to be in radar contact Obey so long as you are not asked to fly below the Minimum Vectoring Altitude in IMC.
Intersection take offs are: * Not allowed for passenger flights Not allowed in Colombia for foreign registered aircraft Allowed under dry pavement conditions only.
When computing weight and balance, the basic empty weight includes the weight of the airframe, engine(s), and all installed optional equipment. Basic empty weight also includes * The unusable fuel, full operating fluids, and full oil All usable fuel, full oil, hydraulic fluid, but does not include the weight of pilot, passengers, or baggage. All usable fuel and oil,but does not include any radio equipment or instruments that were installed by someone other than the manufacturer.
If all index units are positive when computing weight and balance, the location of the datum would be at the* Centerline of the main wheels. Nose, or out in front of the airplane. Centerline of the nose or tailwheel, depending on the type of airplane.
The CG of an aircraft can be determined by which of the following methods?* Dividing total arms by total moments. Multiplying total arms by total weight Dividing total moments by total weight.
Automated flight decks or cockpits* Enhance basic pilot flight skills. Decrease the work load in terminal areas. Often create much larger pilot errors than traditional cockpits.
Identify REIL.* Amber lights for the first 2,000 feet of runway. Green lights at the threshold and red lights at far end of the runway. Synchronized flashing lights laterally at each side of the runway threshold.
What is the advantage of HIRL or MIRL on an IFR runway as compared to a VFR runway?* Lights are closed together and easily distinguinished from surrounding lights. Amber lights replace white on the last 2,000 feet of runway for caution zone. Alternate red and white lights replace the white on the last 3,000 feet of the runway for caution zone.
What does the Precision Approach Path Indicator (PAPI) consist of?* Row of four lights parallel to the runway; red, white and green. Row of four lights perpendicular to the runway; red and white. One light projector with two colors; red and white.
You have just landed at JFK and the tower tells you to call ground control when clear of the runway. You are considered clear of the runway when* The aft end of the aircraft is even with the taxiway location sign. The flight deck area of the aircraft is even with the hold line. All parts of the aircraft hace crossed the hold line.
Refer to Figure 15.) Rwy 30 is being used for landing. Which surface wind would exceed the airplane’s crosswind capability of 0.2 V(SO), if V(SO) is 60 knots? * 260° at 20 knots. 275° at 25 knots. 315° at 35 knots.
(Refer to Figure 15.) If the tower-reported surface wind is 010° at 18 knots, what is the crosswind component for a Rwy 08 landing? * 7 knots. 15 knots. 17 knots.
(Refer to Figure 15.) The surface wind is 180° at 25 knots. What is the crosswind component for a Rwy 13 landing? * 19 knots. 21 knots. 23 knots.
What is the standard temperature at 10,000 feet?* -5°C. -15°C. +5°C.
What is the standard temperature at 20,000 feet?* -15°C. -20°C. -25°C.
W hat are the standard temperature and pressure values for sea level?* 15°C and 29.92" Hg. 59°F and 1013.2" Hg. 15°C and 29.92 Mb.
The performance tables of an aircraft for takeoff and climb are based on* Pressure/density altitude. Cabin altitude. True altitude.
What effect, if any, would a change in ambient temperature or air density have on gas turbine engine performance?* As air density decreases, thrust increases. As temperature increases, thrust increases. As temperature increases, thrust decreases.
With regard to the technique required for a crosswind correction on takeoff, a pilot should use* Aileron pressure into the wind and initiate the lift-off at a normal airspeed in both tailwheel and nose wheel-type airplanes. Right rudder pressure, aileron pressure into the wind, and higher than normal lift-off airspeed in both tricycle- and conventional-gear airplanes. Rudder as required to maintain directional control,aileron pressure into the wind,and higher than normal lift-off airspeed in both conventional- and nosewheel-type airplanes.
When turbulence is encountered during the approach to a landing, what action is recommended and for what primary reason?* Increase the airspeed slightly above normal approach speed to attain more positive control. Decrease the airspeed slightly below normal approach speed to avoid overstressing the airplane. Increase the airspeed slightly above normal approach speed to penetrate the turbulence as quickly as possible.
Pilot’s most immediate and vital concern in the event of complete engine failure after becoming airborne on takeoff is * Maintaining a safe airspeed. Landing directly into the wind. Turning back to the takeoff field.
Which type of approach and landing is recommended during gusty wind conditions?* A power-on approach and power-on landing. A power-off approach and power-on landing. A power-on approach and power-off landing.
A proper crosswind landing on a runway requires that, at the moment of touchdown, the* Direction of motion of the airplane and its lateral axis be perpendicular to the runway. Direction of motion of the airplane and its longitudinal axis be parallel to the runway. Downwind wing be lowered sufficiently to eliminate the tendency for the airplane to drift.
What effect does an uphill runway slope have on takeoff performance?* Increases takeoff speed. Increases takeoff distance. Decreases takeoff distance.
At higher elevation airports the pilot should know that indicated airspeed* Will be unchanged, but groundspeed will be faster. Will be higher, but groundspeed will be unchanged. Should be increased to compensate for the thinner air.
Which condition would INITIALLY cause the indicated airspeed and pitch to increase and the sink rate to decrease? * Sudden decrease in the headwind component. Tailwind which suddenly increases in velocity. Sudden increase in a headwind component.
Which INITIAL cockpit indications should a pilot be aware of when a headwind shears to a calm wind? * Indicated airspeed decreases, aircraft pitches up, and altitude decreases. Indicated airspeed increases, aircraft pitches down, and altitude increases. Indicated airspeed decreases, aircraft pitches down, and altitude decreases.
Which wind-shear condition results in an increase in airspeed? * Increasing tailwind and decreasing headwind. Increasing tailwind and headwind. Decreasing tailwind and increasing headwind.
Which wind-shear condition results in a loss of airspeed? * Decreasing headwind or tailwind. Decreasing headwind and increasing tailwind. Increasing headwind and decreasing tailwind.
What is the recommended technique to counter a loss of airspeed and resultant lift from wind shear? * Lower the pitch attitude and regain lost airspeed. Avoid overstressing the aircraft, "pitch to airspeed", and apply maximum power. Maintain, or increase, pitch attitude and accept the lower-than-normal airspeed indications.
What is the expected duration of an individual microburst? * Two minutes with maximum winds lasting approximately 1 minute. One microburst may continue for as long as 2 to 4 hours. Seldom longer than 15 minutes from the time the burst strikes the ground until dissipation.
Under what conditions would clear air turbulence (CAT) most likely be encountered? * When constant pressure charts show a 20-knot isotaches less than 60 NM apart. When constant pressure charts show a 60-knot isotaches less than 20 NM apart. When a sharp trough is moving at a speed less than 20 knots.
Which is the definition of "severe wind shear"? * Any rapid change of horizontal wind shear in excess of 25 knots; vertical shear excepted. Any rapid change in wind direction or velocity which causes airspeed changes greater than 15 knots or vertical speed changes greater tan 500 ft/min Any change of airspeed greater than 20 Knots which is sustained for more than 20 seconds or vertical speed changes in excess of 100ft/min.
What airport condition is reported by the tower when more than one wind condition at different positions on the airport is reported? * Light and variable. Wind shear. Frontal passage.
Which INITIAL cockpit indications should a pilot be awere of when a constant tailwind shears to a calm wind? * Altitude increases; pitch and indicated airspeed decrease. Altitude, pitch, and indicated airspeed decrease. Altitude, pitch, and indicated airspeed increase.
Clear air turbulence (CAT) associated with a mountain wave may extend as fas as * 1,000 miles or more downstream of the mountain. 5,000 feet above the tropopause. 100 miles or more upwind of the mountain.
What action is appropriate when encountering the first ripple of reported clear air turbulence (CAT)* Extend flaps to decrease wind loading. Extend gear to provide more drag and increase stability. Adjust airspeed to that recomended for rough air.
If severe turbulence is encountered, which procedure is recommended?* Mantain a constant altitude. Mantain a constant attitude. Mantain constant airspeed and altitude.
Which action is recommended regarding an altitude change to get out of the jet stream turbulence?* Descend if ambient temperature is falling. Descend if ambient temperature is rising. Mantain altitude or course to avoid a possible elongated turbulent area.
Hazardous vortex turbulence that might be encontered behind large aircraft is created only when that aircraft is * Developing lift. Operating at high airspeeds. Using high power settings.
Which flight conditions of large jet airplane create the most severe flight hazard by generating wingtip vortices of the greatest strength? * Heavy, slow, gear and flaps up. Heavy, slow, gear and flaps down. Heavy, fast, gear and flaps down.
What effect would a light crosswind have on the wingtip vortices generated by large airplane that has just taken off? * The upwind vortex will tend to remain on the runway longer than the downwind vortex. A crosswind will rapidly dissipate the strength of both vortices. The downwind vortex will tend to remain on the runway longer than the upwind vortex.
What wind condition prolongs the hazard of the wake turbulence on a landing runway for the longest period of time? * Direct tailwind. Light quartering tailwind. Light quartering headwind.
To avoid the wingtip vortices of a desparting jet airplane during takeoff, the pilot should * Lift off at a point well past the jet airplanes flight path. Climb above and stay upwind of the jet airplanes flight path. Remain below the flight path of the jet airplane.
If you take off behaind a jeavy jet that has just landed, you should plan to lift off * prior to the point where the jet touched down. beyond the point where the jet touched down. at the point where the jet touched down and on the upwind edge of the runway.
What is the pilots responsability for the clearance or instruction readback? * Except for SIDs, read back altitude assignmets, altitude restrictions, and vectors. If the clearance or instruction is understood, and acknowledgement is sufficient. Read back the entire clearance or instruction to confirm the message is understood.
What action should a pilot take when a clearance is received from ATC that appears to be contrary to a regulation?* Read the clearance back in its entirety. Request a clarification from ATC. Do no accept the clearance.
What report should the pilot make at a clearance limit? * Time and altitude/flight level arriving or leaving. Time, altitude/flight level, and expected holding speed. Time, altitude/flight level, and expected holding speed, and inbound leg length.
What action should a pilot take if asked by ARTCC to "VERIFY 9,000" and the flight is actually maintaining 8,000? *X Immediately climb to 9000. Report climbing to 9000. Report maintaining 9000.
Where are position reports required on an IFR flight on airway or routes? * Over all designated compulsory reporting points. Only where specifically requested by ARTCC. When requested to change altitude or advise of weather conditions.
Which reports are required when operating IFR in radar environment? * Position reports,vacating and altitude,unable to climb 500ft/min, and time and altitude reaching a holding fix or point to which cleared. Position reports, vacating and altitude, unable to climb 500 ft/min, and time and altitude reaching a holding fix or point to which cleared, and a change in average true airspeed exceeding 5 percent or 10 Knots. Vacating and altitude, unable to climb 500ft/min, time and altitude reaching a holding fix or point to which cleared, a change in average true airspeed exceeding 5 percent or 10 Knots, and leaving and assigned holding fix or point.
TCAS I provides * traffic and resolution advisories. proximity warning. recomended maneuvers to avoid conflicting traffic.
El número de las naciones unidas asignado por el comité de expertos en transporte de MP consta de: *+ 2 dígitos. 3 dígitos. 4 dígitos.
¿Cuántas clases de MP existen? * 9 15 6.
Un animal vivo infectado:* Podrá llevarse abordo debidamente encerrado. Es prohibido transportarlo por vía aérea. Esta prohibido transportarlos por vía aérea salvo dispensa de los estados interesados.
En las normas relacionadas con MP además de los idiomas exigidos por el estado de origen, debería utilizarse: * Ruso Español. Inglés.
Ningún explotador aceptara para su transporte por vía aérea bulto o embalaje externo que contenga MP. * A menos que se haya certificado que los bultos satisfacen las condiciones pertinentes previstas en las instrucciones técnicas Hasta que no se haya inspeccionado que llevan las marcas y etiquetas debidas. Todas las anteriores.
Quien ha instituido procedimientos internacionales que regulan la introducción de MP en el transporte aéreo a través del servicio postal. *+ La OACI. IFALPA. La unión postal universal.
Todo artículo que contiene una o mas sustancias explosivas se denomina: * Incompatible. Artículo explosivo. Liquido pirofórico.
Las sustancias tóxicas se pueden clasificar en: * Sustancias venenosas. Sustancias explosivas Sustancias corrosivas.
Que mercancías no se estibaran junto a otras en una aeronave: * MP incompatibles. Mercancías capaces de reaccionar peligrosamente entre si. A y B son correctas.
Todo bulto de MP llevara las etiquetas de manipulación apropiadas: * Siempre. Vuelos nacionales Vuelos internacionales.
El piloto al mando puede negarse a efectuar un transporte de MP si: *+ No se le ha proporcionado por escrito la información adecuada. No se le ha solicitado la autorización oportuna. El avión lleva VIP. abordo.
Diga que frase es cierta: * Las discrepancias de los estados se comunican a OACI. Las diferencias de los explotadores se comunican a OACI. Las discrepancias de los explotadores se comunican a OACI.
¿La sigla NOTOC que significa? * Notice to company. Notice to crew. Notice to Captain.
¿Si desea buscar una sustancia por su nombre y hallar sus propiedades, donde la buscaría?* Manual de la aeronave. Páginas azules de manual de regulaciones de MP de IATA. En el manual de operaciones de vuelo.
Que significa la sigla DL: * Dosis letal. Dosis dosificada. Dosis limite.
¿Quien asigna el grupo de embalaje? * El expedidor. La IATA. El explotador.
En Las variaciones de los estados que significa la ultima letra que es una G: * Gol Gobierno. Ganancia de cada estado en cuanto MP se refiere.
La letra Y en las instrucciones de embalaje significa: * Que se puede transportar igual cantidad que en una instrucción que no tenga esa letra. La cantidad que se puede transportar es menor porque no es un embalaje aprobado para el transporte de MP. De acuerdo al estado se le pone la letra Y o la Z.
Un embalaje puede ser reutilizado siempre y cuando: * Sean removidas todas las etiquetas. La sustancia a empacar no vaya a reaccionar con algún componente Anterior. A y B son correctas.
TCAS II provides *+ traffic and resolution advisories. proximity warning. maneuvers in all directions to avoid the conflicting traffic.
Each pilot who deviates from an ATC clearance in response to a TCAS advisory is expected to notify ATC and*+ Mantain the course and altitude resulting from the deviation, as ATC has radar contact. Request a new ATC clearance. expeditiously return to the ATC clearance in effect prior to the advisory, after the conflict is resolved.
Each pilot who deviates from an ATC clearance in response to a TCAS II, resolution advisory (RA) is expected to * Maintain the course and altitude resulting from the deviation, as ATC has radar contact. Request a new ATC clearance. notify ATC of the deviation as soon as practicable.
With no traffic identified by TCAS, you * can rest assured that no other aircraft are in the area must continually scan for other traffic in visual conditions. must scan only for hot air balloons.
While being radar vectored, to the final approach course of an IFR approach, when may the pilot descend to published altitudes? *+ Anytime the flight is on a published leg of an approach chart. When the flight is within the 10-mile ring of a published approach. Only when approach control clears the flight for the approach.
What action (s) should a pilot take if vectored across the final approach course during an IFR approach? * Continue on the last heading issued until otherwise instructed. Contact approach control, and advise that the flight is crossing the final approach course. Turn onto final, and broadcast in the blind that the flight has proceeded on final.
What is the difference between a visual and a contact approach? * A visual approach is an IFR authorization while a contact approach is an VFR authorization. A visual approach is initiated by ATC while a contact approach is initiated by the pilot. Both are the same but classified according to the part initiating the approach.
How should a pilot describe braking action? *+ 00 percent, 50 percent, 75 percent, or 100 percent. Zero-zero, fifty-fift, or normal. Nil, poor, fair or good.
When entering a holding pattern above 14,000 feet, the initial outbound leg should not exceed*+ 1 minute. 1-1/2 minutes. 1-1/2 minutes or 10 NM, whichever is less.
Under what condition should a pilot on IFR advise ATC of minimum fuel status? * When the fuel supply becomes less than that required for IFR. If the remaining fuel suggests a need for traffic or landing priority. If the remaining fuel precludes any undue delay.
You should advise ATC of minimum fuel status when your fuel supply has reached a state where, upon reaching your destination, you cannot accept any undue delay. * This will ensure your priority handling by ATC. ATC will consider this action as if you had declared an emergency. If your remaining usable fuel supply suggests the need for traffic priority to ensure a safe landing, declare an emergency due to low fuel and report fuel remaining in minutes.
What is the hijack code? * 7200 7500 7777.
What minimum condition is suggested for declaring an emergency? * Anytime the pilot is doubtful of a condition that could adversely affect flight safety. When fuel endurance or weather will require an en route or landing priority. When distress conditions such as fire, mechanical failure, or structural damage occurs.
Wich range of codes should a pilot avoid switching through when changing transponder codes? * 0000 through 1000. 7200 and 7500 series. 7500, 7600 and 7700 series.
How can the pilot increase the rate of turn and decrease the radius at the same time? * Steepen the bank and increase airspeed. Steepen the bank and decrease airspeed. Shallow the bank and increase airspeed.
Why must the angle of attack be increased during a turn to mantain altitude? * Compensate for loss of vertical component of the lift. Increase the horizontal component of the lift equal to the vertical component. Compensate for increase in drag.
Which condition reduces the required runway for the takeoff? * Higher-than-recomended airspeed before rotation. Lower-than-standard air density. Increase headwind component.
Which performance factor decreases as airplane gross weight increases, for a given runway? * Critical engine failure speed. Rotation speed. Accelerate-stop distance.
What effect does landing at high elevation airports have on groundspeed with comparable conditions relative to temperature, wind and airplane weight? * Higher than at low elevation. Lower than at low elevation. The same as at low elevation.
When checking the course sensitivity of a VOR receiver, how many degrees should the OBS be rotated to move the CDI from the center to the last dot on either side? * 5° to 10°. 10° to 12°. 18° to 20°.
An aircraft 60 miles from a VOR station has a CDI indication of one-fifth deflection, this represents a course centerline deviation of approximately * 6 miles. 2 miles. 1 mile.
Which situation would result in reverse sensing of a VOR receiver? * Flying a heading that is reciprocal to the bearing selected on the OBS. Setting the OBS to a bearing that is 90° from the bearing on which the aircraft is located. Failing to change the OBS from the selected inbound course to the outbound course after passing the station.
To track outbound on the 180 radial of a VOR station, the recommended procedure is to set the OBS to * 360° and make heading corrections toward the CDI needle. 180° and make heading corrections away from the CDI needle. 180° and make heading corrections toward the CDI needle.
To track inbound on the 215 radial of a VOR station, the recommended procedure is to set the OBS to * 215° and make heading corrections toward the CDI needle. 215° and make heading corrections away from the CDI needle. 035° and make heading corrections toward the CDI needle.
What is the primary cause off all changes in the Earths weather? * Variations of solar energy at the Earths surface. Changes in air pressure over the Earth’s surface. Movement of air masses from moist areas to dry areas.
Which weather condition is defined as an anticyclone? X Calm. High pressure area. COL.
Convective clouds which penetrate a stratus layer can produce which threat to instrument flight? * Freezing rain. Clear air turbulence. Embedded thundrestorms.
What phenomenon signals the beginning of the mature stage of a thunderstorm? * The appearance of an anvil top. The start of rain at the surface. Growth rate of cloud is ai its maximum.
What condition is necessary for the formation of structural icing in flight? * Supercooled water drops. Water vapor. Visible water.
The ADF is tuned to a radiobeacon. If the magnetic heading is 040° and the relative bearing is 290°, the magnetic bearing TO that radiobeacon would be * 150°. 285°. 330°.
If the relative bearing to a nondirectional radiobeacon is 045° and the magnetic heading is 355°, the magnetic bearing TO that radio beacon would be * 040°. 065°. 220°.
(Refer to Figure 30.) At the position indicated by instrument group 1, to intercept the 330° magnetic bearing to the NDB at a 30° angle, the aircraft should be turned * Left to a heading of 270°. Right to a heading of 330°. Right to a heading of 360°.
Which type precipitation is an indication that supercooled water is present? * Wet snow. Freezing rain. Ice pellets.
Which weather condition is present when the tropical strom is upgraded to a hurricane? * Highest windspeed, 100 Knots or more. A clear area or hurricane eyes has formed. Sustained winds of 65 Knots or mire.
ILS critical area sign indicates * Where aircraft are prohibited. The edge of the ILS critical area. The exit boundary.
Airport information signs, used to provide destination or information, have * yellow inscriptions on a black background. white inscriptions on a black background. black inscriptions on a yellow background.
Holding position signs have * white inscriptions on a red background. red inscriptions on a white background. yellow inscriptions on a red background.
(Refer to figure 31, point 1) The floor of the Class E airspace above Georgetown Airport (Q61) is at* The surface. 3,823 feet MSL. 700 feet AGL.
Which is true regarding flight operations in Class A airspace? * May conduct operations under visual flight rules. Aircraft must be equipped with approved distance measuring equipment (DME). Aircraft must be equipped with an ATC transponder and altitude reporting equipment.
Which is true regarding flight operations in Class B airspace? * The pilot must receive an ATC clearance before operating an aircraft in that area. Flight under VFR is not authorized unless the pilot in command is instrument rated. Solo student pilot operations are not authorized.
When a dashed blue circle surrounds an airport on a sectional aeronautical chart, it will depict the boundary of * Special VFR airspace. Class D airspace. Class B airspace.
(Refer to figure 32, point 4) The thinner outer magenta circle depicted around San Francisco International Airport is* The outer segment of Class B airspace. An area within which an appropriate transponder must be used from outside of the Class B airspace from the surface to 10,000 feet MSL. A Mode C veil boundary where a balloon may penetrate without a transponder provided it remains below 8,000 feet MSL.
(Refer to figure 31, point 9) The alert area depicted within the blue lines is an area in which* There is a high volume of pilot training activities or an unusual type of aerial activity,neither of which is hazardous to aircraft. The flight of aircraft is prohibited. The flight of aircraft, while not prohibited, is subject to restriction.
Which is true regarding flight operations to or from a satellite airport, without an operating control tower, within the Class C airspace area? * Prior to takeoff, a pilot must establish communication with the ATC controlling facility. Aircraft must be equipped with an ATC transponder. Prior to entering that airspace,a pilot must establish and maintain communication with the ATC serving facility.
(Refer to figure 31, point 7) The floor of Class E airspace over the town of Woodland is* 700 feet AGL over part of the town and no floor over the remainder. 1,200 feet AGL over part of the town and no floor over the remainder. Both 700 feet and 1,200 feet AGL.
Which is true regarding pilot certification requirements for operations in Class B airspace? * The pilot in command must hold at least a private pilot certificate with an instrument rating. The pilot in command must hold at least a private pilot certificate. Solo student pilot operations are not authorized.
(Refer to figure 31, point 5) The floor of the Class E airspace over University Airport (0O5) is * The surface. 700 feet AGL. 1,200 feet AGL.
Which is true regarding flight operations in Class A airspace? * Must conduct operations under instrument flight rules. Aircraft must be equipped with approved distance measuring equipment (DME). Aircraft must be equipped with an approved ATC transponder.
When approaching to land at an airport with an ATC facility, in Class D airspace, the pilot must establish communications prior to * 4 NM, up to and including 2,500 feet AGL. 10 NM, up to and including 3,000 feet AGL. 30 SM, and be transponder equipped.
Refer to figure 14.) While clearing an active runway, you are most likely clear of the ILS critical area when you pass which sign? * Top red. Middle yellow. Bottom yellow.
(Refer to figure 14.) When taxiing up to an active runway, you are likely to be clear of the ILS critical area when short of which symbol?* Top red. Middle yellow. Bottom yellow.
To best overcome the effects of spatial disorientation, a pilot should * Rely on body sensations. Increase the breathing rate. Rely on aircraft instrument indications.
Which would most likely result in hyperventilation? * Insufficient oxygen. Excessive carbon monoxide. Insufficient carbon dioxide.
To overcome the symptoms of hyperventilation, a pilot should * Swallow or yawn. Slow the breathing rate. Increase the breathing rate.
Which is a common symptom of hyperventilation? * Drowsiness. Decreased breathing rate. A sense of well-being.
Which is true regarding the presence of alcohol within the human body? * A small amount of alcohol increases vision acuity. An increase in altitude decreases the adverse effect of alcohol. Judgment and decision-making abilities can be adversely affected by even small amounts of alcohol.
Hypoxia is the result of which of these conditions? * Excessive oxygen in the bloodstream. Insufficient oxygen reaching the brain. Excessive carbon dioxide in the bloodstream.
Hypoxia susceptibility due to inhalation of carbon monoxide increases as * Humidity decreases. Altitude increases. Oxygen demand increases.
Which is true regarding the use of a Instrument Departure Procedure (DP) chart? * At airfields where DP's have been established, DP usage is mandatory for IFR departures. To use a DP, the pilot must possess at least the textual description of the approved standard departure. To use a DP,the pilot must possess both the textual and graphic form of the approved standard departure.
(Refer to figures 33) The final approach fix for the ILS precision approach is located at * DENAY intersection. Glide slope intercept. ROMEN intersection/locator outer marker.
(Refer to figure 34.) In the DEN ILS RWY 35R procedure, In the DEN ILS RWY 35R procedure, the glide slope intercept altitude is * 11,000 feet MSL. 7,000 feet MSL. 9,000 feet MSL.
(Refer to figure 36) When approaching the ATL ILS RWY 8L, how far from the FAF is the missed approach point?* 4.8 NM. 5.2 NM. 12.0 NM.
(Refer to figure 37) If the glide slope becomes inoperative during the ILS RWY 31R procedure at DSM, what MDA applies? * 1,157 feet. 1,320 feet. 1,360 feet.
(Refer to figure 37.) During the ILS RWY 31R procedure at DSM, the minimum altitude for glide slope interception is * 2,365 feet MSL. 2,400 feet MSL. 3,000 feet MSL.
The pilot in command of an aircraft operated under IFR, in controlled airspace, shall report as soon as practical to ATC when* Experiencing any malfunctions of navigational, approach,or communications equipment,occurring in flight. Requested to contact a new controlling facility. Climbing or descending to assigned altitudes.
(Refer to figure 35.) What minimum navigation equipment is required to complete the VOR/DME-A procedure? * One VOR receiver. One VOR receiver and DME. Two VOR receivers and DME.
Which is true regarding STAR's? STAR's are * Used to separate IFR and known VFR traffic. To facilitate transition between en route and instrument approach procedures. Used at certain airports to relieve traffic congestion.
While being radar vectored, an approach clearance is received. The last assigned altitude should be maintained until * Reaching the FAF. Advised to begin descent. Established on a segment of a published route or instrument approach procedure.
(Refer to figure 34.) The symbol [9200] in the MSA circle of the ILS RWY 35R procedure at DEN represents a minimum safe sector altitude within 25 NM of * Denver VORTAC. Dymon outer marker. Cruup I-AQD DME fix.
What does the absence of the procedure turn barb on the plan view on an approach chart indicate? * A procedure turn is not authorized. Teardrop-type procedure turn is authorized. Racetrack-type procedure turn is authorized.
A pilot performing a published instrument approach is not authorized to perform a procedure turn when * Maneuvering at radar vectoring altitudes. Receiving a radar vector to a final approach course or fix. Maneuvering at minimum safe altitudes.
On an instrument approach where a DH or MDA is applicable, the pilot may not operate below, or continue the approach unless the * Flight visibility and ceiling are at, or above, the published minimums for that approach. Approach and runway lights are distinctly visible to the pilot. Aircraft is continuously in a position from which a descent to a normal landing,on the intended runway, can be made.
Pilots are not authorized to land an aircraft from an instrument approach unless the * Flight visibility is at, or exceeds the visibility prescribed in the approach procedure being used. Flight visibility and ceiling are at, or exceeds the minimums prescribed in the approach being used. Visual approach slope indicator and runway references are distincly visible to the pilot.
(Refer to figure 38) During the ILS RWY 13L procedure at DSM, what altitude minimum applies if the glide slope becomes inoperative? * 1,420 feet. 1,340 feet. 1,121 feet.
For an airport without an approved instrument approach procedure to be listed as an alternate airport on an IFR flight plan, the forecasted weather conditions at the time of arrival must have at least a * Ceiling of 2,000 feet and visibility 3 SM. Ceiling and visibility that allows for a descent, approach, and landing under basic VFR. Ceiling of 1,000 feet and visibility 3 NM.
For an airport with an approved instrument approach procedure to be listed as an alternate airport on an IFR flight plan, the forecasted weather conditions at the time of arrival must be at or above the following weather minimums. * Ceiling 800 feet and visibility 2 SM for nonprecision. Ceiling 800 feet and visibility 2 NM for nonprecision. Ceiling 600 feet and visibility 2 NM for precision.
(Refer to figure 32, point 6) The Class C airspace at Metropolitan Oakland International (OAK) which extends from the surface upward has a ceiling of * Both 2,100 feet and 3,000 feet MSL. 8,000 feet MSL. 2,100 feet AGL.
VFR is the same as VMC and IFR is the same as IMC. * True, they are both the same False, one is the type of flight plan and the other the designation as per Ops. Specs. False, but they are correlated.
Hypoxia is the result of: * The brain and body tissue not receiving sufficient CO through the blood stream The brain not being used to a lack of oxygen All of your body not being sufficiently balanced with oxygen and becoming hypoxic.
A DME arc should be flown within ____ from the center of the arc: * 1.0 DME 2.0 DME to either side of the arc 0.0. (No deviations are allowed).
If you are flying at 18.000 Ft. and your DME reads 6.0 Nm, you are: * 6.0 Nm from the VOR 4.5 Nm from the VOR 3.0 Nm from the VOR.
TERPs are a US standard used for: * All ICAO special charts IFR departure design criteria Non Jeppesen stars.
When flying a DP and you lose communications, you must: * Continue flying the DP as published Return for a landing Proceed as per special instructions on the chart when included.
Take off minimums are a standard value or all airports in Colombia: * True, except as published on STARs False, every airport is different True, except where otherwise published on the chart or company Ops Specs.
Departure Procedures must: * Be flown if available for that airport Be flown and no change request for another DP is allowed if on an IFR flight plan Be flown if accepted but may be declined or changes made if coordinated with ATC.
On a Jeppesen or NOA chart what does V344 mean: * V stands for “very high speed” and 344 is the max. speed allowed in Kts for that direction of flight V stands for VOR (VHF signal) and 344 is the airway allocation V stands or “VictorCollins” whom in 1935 invented the airways between VORs and 344 is the airway allocation.
MEA guarantees adequate terrain clearance and: * Navigation signal Both navigation and communications 1.500 ground clearance around mountains.
When flying an airway you need to deviate 3.5 Nm to the left of course you need: * Do nothing, airways are 4 Nm wide to each side Request ATC since you are expected to navigate on the center line of the airway Do nothing, airways are 8 Nm wide to each side of the center line.
1.000 Ft clearance from terrain and 2.000 Ft in mountainous areas is guaranteed on:* MVA, MEA and MOCA MAA, MCA, MRA, MOCA and MEA Neither, as the radio altimeter is never activated (2.500Ft or less) when the aircraft flies in mountainous terrain.
In order to fly a route where you plan to fly with an IFR approved GPS as the primary means of navigation from airport A direct to airport B, you need: * A Hi or Lo enroute chart A GPS navigation chart since airways will not be used A WAC VFR chart since GPS is not certified for enroute navigation.
When departing an airport in class B airspace you should advice ATC upon initial contact once handed off from TWR:* Your altitude assigned level/alt, squak code Aircraft ID, present altitude, squak code Aircraft ID, present altitude, limiting altitude assigned and squak code.
When in radar contact, your aircraft DME equipment becomes unserviceable, you should:* Write a maintenance report at your next stop Advice ATC so he/she if required may make alternate arrangements Check the MEL to ensure the equipment is not required for the type of flight you are executing.
Position reports when required should include:* Aircraft ID, position, ETA over next PT and following point Nothing if you were advised to be in radar contact since in this condition ATL always knows where you are Aircraft ID, position, time, altitude, ETA at next point, following compulsory point, remarks if any.
You are in radar contact and ATC clears you to a lower altitude at pilot’s discretion, you should:* Report leaving present altitude. Just descend at will since ATC will know your altitude at all times Descend as quickly as possible to relieve ATC of your traffic.
You gave a position report and you recognize that your ETA at the following point is off, you should: * Advice ATC if more than 5 % error or 10 miles If off by more than 3 minutes advice ATC of revised time Do nothing as you are on an IFR Flt.
In a no wind condition you are asked at 12:00 Hrs. to hold at 8.000 Ft. until 12:12 Hrs. * You will fly two complete racetrack patterns You will fly three complete racetrack patterns The # of circuits you fly depends on the speed you are holding at.
If on a descent clearance you are cleared to hold at a fix, you should: * Slow down and enter the hold at VMC + 10 Kts Reach the fix at or below the speed for your aircraft type/category Maintain the speed up so as to hold at the fastest speed possible to be the first to leave the fix and expedite ATC.
When is an IFR flight plan required? * When less than VFR conditions exist in either Class E or Class G airspace and in Class A airspace. In all Class E airspace when conditions are below VFR, in Class A airspace, and in defense zone airspace. In Class E airspace when IMC exists or in Class A airspace.
Prior to which operation must an IFR flight plan be filed and an appropriate ATC clearance received? * Flying by reference to instruments in controlled airspace. Entering controlled airspace when IMC exists. Takeoff when IFR weather conditions exist.
To operate under IFR below 18,000 feet, a pilot must file an IFR flight plan and receive an appropriate ATC clearance prior to * Entering controlled airspace. Entering weather conditions below VFR minimums. Takeoff.
To operate an aircraft under IFR, a flight plan must have been filed and an ATC clearance received prior to * Controlling the aircraft solely by use of instruments. Entering weather conditions in any airspace. Entering controlled airspace.
When is an IFR clearance required during VFR weather conditions? * When operating in the Class E airspace. When operating in the Class A airspace. When operating in airspace above 14,500 feet.
When departing from an airport located outside controlled airspace during IMC, you must file an IFR flight plan and receive a clearance before * Takeoff. Entering IFR conditions. Entering Class E airspace.
No person may operate an aircraft in controlled airspace under IFR unless he/she files a flight plan * And receives a clearance by telephone prior to takeoff. Prior to takeoff and requests the clearance upon arrival on an airway. And receives a clearance prior to entering controlled airspace.
During your preflight planning for an IFR flight, you determine that the first airport of intended landing has no instrument approach prescribed in 14 CFR part 97. The weather forecast for one hour before through one hour after your estimated time of arrival is 3000' scattered with 5 miles visibility. To meet the fuel requirements for this flight, you must be able to fly to the first airport of intended landing: * Then to the alternate airport, and then for 30 minutes at normal cruising speed. Then to the alternate airport, and then for 45 minutes at normal cruising speed. And then fly for 45 minutes at normal cruising speed.
What are the minimum fuel requirements in IFR conditions, if the first airport of intended landing is forecast to have a 1,500-foot ceiling and 3 miles visibility at flight-planned ETA? Fuel to fly to the first airport of intended landing * And fly thereafter for 45 minutes at normal cruising speed. Fly to the alternate, and fly thereafter for 45 minutes at normal cruising speed. Fly to the alternate, and fly thereafter for 30 minutes at normal cruising speed.
What minimum weather conditions must be forecast for your ETA at an alternate airport, that has only a VOR approach with standard alternate minimums, for the airport to be listed as an alternate on the IFR flight plan? * 800-foot ceiling and 1 statute mile visibility. 800-foot ceiling and 2 statute miles visibility. 1,000-foot ceiling and visibility to allow descent from minimum enroute altitude(MEA),approach,and landing under basic VFR.
What are the alternate minimums that must be forecast at the ETA for an airport that has a precision approach procedure? * 400-foot ceiling and 2 miles visibility. 600-foot ceiling and 2 miles visibility. 800-foot ceiling and 2 miles visibility.
When an alternate airport is required, what are the weather minimums that must be forecast at the ETA for an alternate airport that has a precision approach procedure?* Ceiling 200 feet above the approach minimums and at least 1 statute mile visibility, but not less than the minimum visibility for the approach. 600 foot ceiling and 2 statute miles visibility. Ceiling 200 feet above field elevation and visibility 1 statute mile, but not less than the minimum visibility for the approach.
What are the minimum weather conditions that must be forecast to list an airport as an alternate when the airport has no approved IAP? * The ceiling and visibility at ETA, 2,000 feet and 3 miles, respectively. The ceiling and visibility from 2 hours before until 2 hours after ETA, 2,000 feet and 3 miles, respectively. The ceiling and visibility at ETA must allow descent from MEA, approach, and landing, under basic VFR.
When a pilot elects to proceed to the selected alternate airport, which minimums apply for landing at the alternate? * 600-1 if the airport has an ILS. Ceiling 200 feet above the published minimum, visibility 2 miles. The landing minimums for the approach to be used.
If a pilot elects to proceed to the selected alternate, the landing minimums used at that airport should be the * Minimums specified for the approach procedure selected. Alternate minimums shown on the approach chart. Minimums shown for that airport in a separate listing of “IFR Alternate Minimums.”.
When making an instrument approach at the selected alternate airport, what landing minimums apply? * Standard alternate minimums (600-2 or 800-2). The IFR alternate minimums listed for that airport. The landing minimums published for the type of procedure selected.
An airport without an authorized IAP may be included on an IFR flight plan as an alternate, if the current weather forecast indicates that the ceiling and visibility at the ETA will * Allow for descent from the IAF to landing under basic VFR conditions. Be at least 1,000 feet and 1 mile. Allow for a descent from the MEA, approach, and a landing under basic VFR conditions.
Preferred IFR routes beginning with a fix indicate that departing aircraft will normally be routed to the fix by* The established airway(s) between the departure airport and the fix. An instrument departure procedure (DP), or radar vectors. Direct route only.
The most current en route and destination flight information for planning an instrument flight should be obtained from* The ATIS broadcast. The FSS. Notices to Airmen (Class II).
What point at the destination should be used to compute estimated time en route on an IFR flight plan? * The final approach fix on the expected instrument approach. The initial approach fix on the expected instrument approach. The point of first intended landing.
For aircraft other than helicopters, what forecast weather minimums are required to list an airport as an alternate on an IFR flight plan if the airport has VOR approach only?* Ceiling and visibility at ETA, 800 feet and 2 miles, respectively. Ceiling and visibility from 2 hours before until 2 hours after ETA, 800 feet and 2 miles, respectively Ceiling and visibility at ETA, 600 feet and 2 miles, respectively.
For aircraft other than helicopters, what minimum weather conditions must be forecast for your ETA at an alternate airport that has a precision approach procedure, with standard alternate minimums, in order to list it as an alternate for the IFR flight?* 600-foot ceiling and 2 SM visibility at your ETA. 600-foot ceiling and 2 SM visibility from 2 hours before to 2 hours after your ETA. 800-foot ceiling and 2 SM visibility at your ETA.
For aircraft other than helicopters, is an alternate airport required for an IFR flight to ATL (Atlanta Hartsfield) if the proposed ETA is 1930Z?, TAF KATL 121720Z 121818 20012KT 5SM HZ BKN030 FM2000 3SM TSRA OVC025CB FM2200 33015G20KT P6SM BKN015 OVC040 BECMG 0608 02008KT BKN 040 BECMG 1012 00000KT P6SM CLR= * No, because the ceiling and visibility are forecast to be at or above 2,000 feet and 3 miles within 1 hour before to 1 hour after the ETA. No, because the ceiling and visibility are forecast to remain at or above 1,000 feet and 3 miles, respectively. Yes, because the ceiling could fall below 2,000 feet within 2 hours before to 2 hours after the ETA.
For aircraft other than helicopters, what minimum conditions must exist at the destination airport to avoid listing an alternate airport on an IFR flight plan when a standard IAP is available? * From 2 hours before to 2 hours after ETA, forecast ceiling 2,000, and visibility 2 and 1/2 miles. From 2 hours before to 2 hours after ETA, forecast ceiling 3,000, and visibility 3 miles. From 1 hour before to 1 hour after ETA, forecast ceiling 2,000, and visibility 3 miles.
What action is recommended if a pilot does not wish to use an instrument departure procedure? * Advise clearance delivery or ground control before departure. Advise departure control upon initial contact. Enter “No DP” in the REMARKS section of the IFR flight plan.
Reception of signals from a radio facility, located off the airway being flown, may be inadequate at the designated MEA to identify the fix. In this case, which altitude is designated for the fix? * MOCA. MRA. MCA.
ATC may assign the MOCA when certain special conditions exist, and when within * 22 NM of a VOR. 25 NM of a VOR. 30 NM of a VOR.
For IFR operations off of established airways below 18,000 feet, VOR navigation aids used to describe the “route of flight” should be * 40 NM apart. 70 NM apart. 80 NM apart.
At what point should the timing begin for the first leg outbound in a nonstandard holding pattern? * Abeam the holding fix, or wings level, whichever occurs last. When the wings are level at the completion of the 180° turn outbound. When over or abeam the holding fix, whichever occurs later.
MEA is an altitude which assures * Obstacle clearance, accurate navigational signals from more than one VORTAC,and accurate DME mileage A 1,000-foot obstacle clearance within 2 miles of an airway and assures accurate DME mileage. Acceptable navigational signal coverage and meets obstruction clearance requirements.
To ensure proper airspace protection while in a holding pattern, what is the maximum airspeed above 14,000 feet for civil turbojet aircraft? * 230 knots. 265 knots. 200 knots.
What obstacle clearance and navigation signal coverage is a pilot assured with the Minimum Sector Altitudes depicted on the IAP charts? * 1,000 feet and acceptable navigation signal coverage within a 25 NM radius of the navigation facility. 1,000 feet within a 25 NM radius of the navigation facility but not acceptable navigation signal coverage. 500 feet and acceptable navigation signal coverage within a 10 NM radius of the navigation facility.
When holding at an NDB, at what point should the timing begin for the second leg outbound? * When the wings are level and the wind drift correction angle is established after completing the turn to the outbound heading. When the wings are level after completing the turn to the outbound heading, or abeam the fix, whichever occurs first. When abeam the holding fix.
(Refer to Figures 39 and 40.) What is the significance of the symbol at GRICE intersection?* It signifies a localizer-only approach is available at Harry P. Williams Memorial. The localizer has an additional navigation function. GRICE intersection also serves as the FAF for the ILS approach procedure to Harry P. Williams Memorial.
To ensure proper airspace protection while in a holding pattern, what is the maximum indicated airspeed above 14,000 feet? * 220 knots. 265 knots. 200 knots.
What is the definition of MEA? * The lowest published altitude which meets obstacle clearance requirements and assures acceptable navigational signal coverage. The lowest published altitude which meets obstacle requirements, assures acceptable navigational signal coverage, two-way radio communications, and provides adequate radar coverage. An altitude which meets obstacle clearance requirements,assures acceptable navigation signal coverage, two-way radio communications, adequate radar coverage, and accurate DME mileage.
To ensure proper airspace protection while holding at 5,000 feet in a civil aircraft, what is the maximum indicated airspeed a pilot should use? * 230 knots. 200 knots. 210 knots.
What timing procedure should be used when performing a holding pattern at a VOR? * Timing for the outbound leg begins over or abeam the VOR, whichever occurs later. Timing for the inbound leg begins when initiating the turn inbound. Adjustments in timing of each pattern should be made on the inbound leg.
Acceptable navigational signal coverage at the MOCA is assured for a distance from the VOR of only * 12 NM. 22 NM. 25 NM.
Which aeronautical chart depicts Military Training Routes (MTR) above 1,500 feet? * IFR Planning Chart. IFR Low Altitude En Route Chart. IFR High Altitude En Route Chart.
When more than one circuit of the holding pattern is needed to lose altitude or become better established on course, the additional circuits can be made * At pilot’s discretion. Only in an emergency. Only if pilot advises ATC and ATC approves.
(Refer to Figure 41.) En route on V112 from BTG VORTAC to LTJ VORTAC, the minimum altitude crossing Gymme intersection is * 6,400 feet. 6,500 feet. 7,000 feet.
Reception of signals from an off-airway radio facility may be inadequate to identify the fix at the designated MEA. In this case, which altitude is designated for the fix? * MRA MCA MOCA.
Which condition is guaranteed for all of the following altitude limits: MAA, MCA, MRA, MOCA, and MEA? (Non-mountainous area.) * Adequate navigation signals. Adequate communications. 1,000-foot obstacle clearance.
If no MCA is specified, what is the lowest altitude for crossing a radio fix, beyond which a higher minimum applies? * The MEA at which the fix is approached. The MRA at which the fix is approached. The MOCA for the route segment beyond the fix.
(Refer to Figure 42.) Which VHF frequencies, other than 121.5, can be used to receive De Ridder FSS in the Lake Charles area? * 122.1, 126.4. 123.6, 122.65 122.2, 122.3.
How can an IAF be identified on a Standard Instrument Approach Procedure (SIAP) Chart?* All fixes that are labeled IAF. Any fix illustrated within the 10 mile ring other than the FAF or stepdown fix. The procedure turn and the fixes on the feeder facility ring.
(Refer to Figures 43 and 43A.) What is the difference in elevation (in feet MSL) between the airport elevation and the TDZE for RWY 36L? * 15 feet. 18 feet. 22 feet.
(Refer to Figure 44.) What is the minimum altitude at which you should intercept the glide slope on the ILS RWY 6 approach procedure? * 3,000 feet MSL. 1,800 feet MSL. 1,690 feet MSL.
When cleared to execute a published sidestep maneuver for a specific approach and landing on the parallel runway, at what point is the pilot expected to commence this maneuver?* At the published minimum altitude for a circling approach. As soon as possible after the runway or runway environment is in sight. At the localizer MDA minimum and when the runway is in sight.
Aircraft approach categories are based on * Certificated approach speed at maximum gross weight. 1.3 times the stall speed in landing configuration at maximum gross landing weight. 1.3 times the stall speed at maximum gross weight.
(Refer to Figure 45.) If cleared for NDB RWY 28 approach (Lancaster/Fairfield) over ZZV VOR, the flight would be expected to Category A aircraft Last assigned altitude 3,000 feet* Proceed straight in from CRISY, descending to MDA after CASER. Proceed to CRISY, then execute the teardrop procedure as depicted on the approach chart. Proceed direct to CASER, then straight in to S-28 minimums of 1620-1.
If all ILS components are operating and the required visual references are not established, the missed approach should be initiated upon * Arrival at the DH on the glide slope. Arrival at the middle marker. Expiration of the time listed on the approach chart for missed approach.
(Refer to Figure 46.) What is the purpose of the 10,300 MSA on the Price/Carbon County Airport Approach Chart? * It provides safe clearance above the highest obstacle in the defined sector out to 25 NM. It provides an altitude above which navigational course guidance is assured. It is the minimum vector altitude for radar vectors in the sector southeast of PUC between 020° and 290° magnetic bearing to PUC VOR.
(Refer to Figure 43A.) Which navigational information and services would be available to the pilot when using the localizer frequency? * Localizer and glide slope, DME, TACAN with no voice capability. Localizer information only, ATIS and DME are available. Localizer and glide slope, DME, and no voice capability.
(Refer to Figures 47 and 48.) Which aircraft approach category should be used for a circling approach for a landing on RWY 27?* A B C.
Under which condition does ATC issue a STAR? * To all pilots wherever STAR’s are available. Only if the pilot requests a STAR in the “Remarks” section of the flight plan. When ATC deems it appropriate, unless the pilot requests “No STAR.”.
(Refer to Figure 49.) Using an average ground speed of 90 knots, what constant rate of descent from 2,400 feet MSL at the 6 DME fix would enable the aircraft to arrive at 2,000 feet MSL at the FAF?* 200 feet per minute. 400 feet per minute. 600 feet per minute.
(Refer to Figure 50.) If cleared for a straight-in LOC approach from over OALDY, it means the flight should * Land straight in on runway 31. Comply with straight-in landing minimums. Begin final approach without making a procedure turn.
(Refer to Figure 51.) The symbol on the plan view of the ILS RWY 35R procedure at DEN represents a minimum safe sector altitude within 25 NM of * Denver VORTAC. Gandi outer marker. Denver/Stapleton International Airport.
Where a holding pattern is specified in lieu of a procedure turn, the holding maneuver must be executed within * The 1 minute time limitation or DME distance as specified in the profile view. A radius of 5 miles from the holding fix. 10 knots of the specified holding speed.
Which procedure should be followed by a pilot who is circling to land in a Category B airplane, but is maintaining a speed 5 knots faster than the maximum specified for that category? * Use the approach minimums appropriate for Category C. Use Category B minimums. Use Category D minimums since they apply to all circling approaches.
When the approach procedure involves a procedure turn, the maximum speed should not be greater than * 180 knots IAS. 200 knots IAS. 250 knots IAS.
What does the symbol T within a black triangle in the minimums section of the IAP for a particular airport indicate? * Take off minimums are 1 mile for aircraft having two engines or less and 1/2 mile for those with more than two engines Instrument takeoffs are not authorized. Takeoff minimums are not standard and/or departure procedures are published.
When simultaneous approaches are in progress, how does each pilot receive radar advisories? * On tower frequency. On approach control frequency. One pilot on tower frequency and the other on approach control frequency.
When may a pilot make a straight-in landing, if using an IAP having only circling minimums?* A straight-in landing may not be made, but the pilot may continue to the runway at MDA and then circle to land on the runway. The pilot may land straight-in if the runway is the active runway and he has been cleared to land. A straight-in landing may be made if the pilot has the runway in sight in sufficient time to make a normal approach for landing, and has been cleared to land.
What does the absence of the procedure turn barb on the plan view on an approach chart indicate? * A procedure turn is not authorized. Teardrop-type procedure turn is authorized. Racetrack-type procedure turn is authorized.
(Refer to Figure 43A.) What rate of descent should you plan to use initially to establish the glidepath for the ILS RWY 36L approach? (Use 120 knots ground speed.) * 425 feet per minute. 530 feet per minute. 635 feet per minute.
Which fixes on the IAP Charts are initial approach fixes? * Any fix on the en route facilities ring, the feeder facilities ring, and those at the start of arc approaches. Only the fixes at the start of arc approaches and those on either the feeder facilities ring or en route facilities ring that have a transition course shown to the approach procedure. Any fix that is identified by the letters IAF.
(Refer to Figure 52.) What is the elevation of the TDZE for RWY 4? * 70 feet MSL. 54 feet MSL. 46 feet MSL.
(Refer to Figure 53.) What is the TDZ elevation for RWY 16 on Eugene/Mahlon Sweet Field?* 363 feet MSL. 365 feet MSL. 396 feet MSL.
What does the Runway Visual Range (RVR) value, depicted on certain straight-in IAP Charts, represent? * The slant range distance the pilot can see down the runway while crossing the threshold on glide slope. The horizontal distance a pilot should see when looking down the runway from a moving aircraft. The slant visual range a pilot should see down the final approach and during landing.
(Refer to Figure 53) Using a ground speed of 90 knots on the ILS final approach course, what rate of descent should be used as a reference to maintain the ILS glide slope? * 415 feet per minute. 480 feet per minute. 555 feet per minute.
(Refer to Figure 54.) With a ground speed of 120 knots, approximately what minimum rate of descent will be required between I-GPO 7 DME fix (ROBOT) and the I-GPO 4 DME fix? * 1,200 fpm. 500 fpm. 800 fpm.
What conditions are necessary before ATC can authorize a visual approach? * You must have the preceding aircraft in sight, and be able to remain in VFR weather conditions. You must have the airport in sight or the preceding aircraft in sight, and be able to proceed to, and land in IFR conditions. You must have the airport in sight or a preceding aircraft to be followed,and be able to proceed to the airport in VFR conditions.
Assume this clearance is received: “CLEARED FOR ILS RUNWAY 07 LEFT APPROACH, SIDE-STEP TO RUNWAY 07 RIGHT.” When would the pilot be expected to commence the side-step maneuver? * As soon as possible after the runway environment is in sight Any time after becoming aligned with the final approach course of Runway 07 left, and after passing the final approach fix. After reaching the circling minimums for Runway 07 right.
(Refer to Figure 55.) What indication should you get when it is time to turn inbound while in the procedure turn at LABER? * 4 DME miles from LABER. 10 DME miles from the MAP. 12 DME miles from LIT VORTAC.
(Refer to Figure 54.) What is the usable runway length for landing on runway 21 at PDX?* 5,957 feet. 7,000 feet. 7,900 feet.
If the RVR equipment is inoperative for an IAP that requires a visibility of 2,400 RVR, how should the pilot expect the visibility requirement to be reported in lieu of the published RVR? * As a slant range visibility of 2,400 feet. As an RVR of 2,400 feet. As a ground visibility of 1/2 SM.
(Refer to Figure 56.) Under which condition should the missed approach procedure for the VOR/DME RNAV RWY 33 approach be initiated? * Immediately upon reaching the 5.0 DME from the FAF. When passage of the MAP waypoint is shown on the ambiguity indicator. After the MDA is reached and 1.8 DME fix from the MAP waypoint.
Which of the following statements is true regarding Parallel ILS approaches? * Parallel ILS approach runway centerlines are separated by at least 4,300 feet and standard IFRseparation is provided on the adjacent runway. Parallel ILS approaches provide aircraft a minimum of 1-1/2 miles radar separation between successive aircraft on the adjacent localizer course. Landing minimums to the adjacent runway will be higher than the minimums to the primary runway,but will normally be lower than the published circling minimums.
The RVR minimums for takeoff or landing are published in an IAP, but RVR is inoperative and cannot be reported for the runway at the time. Which of the following would apply? RVR minimums which are specified in the procedure should be converted and applied as ground visibility. * RVR minimums may be disregarded, providing the runway has an operative HIRL system. RVR minimums may be disregarded, providing all other components of the ILS system are operative.
If an early missed approach is initiated before reaching the MAP, the following procedure should be used unless otherwise cleared by ATC. * Proceed to the missed approach point at or above the MDA or DH before executing a turning maneuver Begin a climbing turn immediately and follow missed approach procedures. Maintain altitude and continue past MAP for 1 minute or 1 mile whichever occurs first.
(Refer to Figure 54.) You have been cleared to the CREAK intersection via the BTG 054° radial at 7,000 feet. Approaching CREAK, you are cleared for the LOC/DME RWY 21 approach to PDX. Descent to procedure turn altitude should not begin prior to* completion of the procedure turn, and established on the localizer. CREAK outbound. intercepting the glide slope.
(Refer to Figure 57.) If your aircraft was cleared for the ILS RWY 17R at Lincoln Municipal and crossed the Lincoln VOR at 5,000 feet MSL, at what point in the teardrop could a descent to 3,000 feet commence? * As soon as intercepting LOC inbound. Immediately. Only at the point authorized by ATC.
If during an ILS approach in IFR conditions, the approach lights are not visible upon arrival at the DH, the pilot is * Required to immediately execute the missed approach procedure. Permitted to continue the approach and descend to the localizer MDA. Permitted to continue the approach to the approach threshold of the ILS runway.
Denunciar test Consentimiento Condiciones de uso